User login
How bariatric surgery affects psychotropic drug absorption
Ms. B, age 60, presents to the clinic with high blood pressure, hyperlipidemia, type 2 diabetes mellitus, depression, and anxiety. Her blood pressure is 138/82 mm Hg and pulse is 70 beats per minute. Her body mass index (BMI) is 41, which indicates she is obese. She has always struggled with her weight and has tried diet and lifestyle modifications, as well as medications, for the past 5 years with no success. Her current medication regimen includes lisinopril 40 mg daily, amlodipine 5 mg daily, atorvastatin 40 mg daily, metformin 500 mg twice daily, dulaglutide 0.75 mg weekly, lithium 600 mg daily, venlafaxine extended-release (XR) 150 mg daily, and alprazolam 0.5 mg as needed up to twice daily. Due to Ms. B’s BMI and because she has ≥1 comorbid health condition, her primary care physician refers her to a gastroenterologist to discuss gastric bypass surgery options.
Ms. B is scheduled for Roux-en-Y gastric bypass surgery. You need to determine if any changes should be made to her psychotropic medications after she undergoes this surgery.
There are multiple types of bariatric surgeries, including Roux-en-Y gastric bypass, sleeve gastrectomy, laparoscopic adjustable gastric band, and biliopancreatic diversion with duodenal switch (BPD/DS) (Figure1-4). These procedures all restrict the stomach’s capacity to hold food. In most cases, they also bypass areas of absorption in the intestine and cause increased secretion of hormones in the gut, including (but not limited to) peptide-YY (PYY) and glucagon-like peptide 1 (GLP-1). These hormonal changes impact several factors, including satiety, hunger, and blood sugar levels.5
Roux-en-Y is commonly referred to as the gold standard of weight loss surgery. It divides the top of the stomach into a smaller stomach pouch that connects directly to the small intestine to facilitate smaller meals and alters the release of gut hormones. Additionally, a segment of the small intestine that normally absorbs nutrients and medications is completely bypassed. In contrast, the sleeve gastrectomy removes approximately 80% of the stomach, consequently reducing the amount of food that can be consumed. The greatest impact of the sleeve gastrectomy procedure appears to result from changes in gut hormones. The adjustable gastric band procedure works by placing a band around the upper portion of the stomach to create a small pouch above the band to satisfy hunger with a smaller amount of food. Lastly, BPD/DS is a procedure that creates a tubular stomach pouch and bypasses a large portion of the small intestine. Like the gastric bypass and sleeve gastrectomy, BPD/DS affects gut hormones impacting hunger, satiety, and blood sugar control.
How bariatric surgery can affect drug absorption
As illustrated in the Table,6-19 each type of bariatric surgery may impact drug absorption differently depending on the mechanism by which the stomach is restricted.
Drug malabsorption is a concern for clinicians with patients who have undergone bariatric surgery. There is limited research measuring changes in psychotropic exposure and outcomes following bariatric surgery. A 2009 literature review by Padwal et al7 found that one-third of the 26 studies evaluated provided evidence of decreased absorption following bariatric surgery in patients taking medications that had intrinsic poor absorption, high lipophilicity, and/or undergo enterohepatic recirculation. In a review that included a small study of patients taking selective serotonin reuptake inhibitors or venlafaxine, Godini et al8 demonstrated that although there was a notable decrease in drug absorption closely following the surgery, drug absorption recovered for some patients 1 month after Roux-en-Y surgery. These reviews suggest patients who have undergone any form of bariatric surgery must be observed closely because drug absorption may vary based on the individual, the medication administered, and the amount of time postprocedure.
Until more research becomes available, current evidence supports recommendations to assist patients who have a decreased ability to absorb medications after gastric bypass surgery by switching from an extended-release formulation to an immediate-release or solution formulation. This allows patients to rely less on gastric mixing and unpredictable changes in drug release from extended- or controlled-release formulations.
Continue to: Aside from altered...
Aside from altered pharmacokinetics after bariatric surgery, many patients experience an increased risk of self-harm and suicide.20 Therefore, a continued emphasis on and reinforcement of proper antidepressant use and adjustment in these patients is important. This can be facilitated through frequent follow-up visits, either in-person or via telehealth.
Understanding the effect of bariatric surgery on drug absorption is critical to identifying a potential need to adjust a medication dose or formulation after the surgery. Available evidence and data suggest it is reasonable to switch from an extended- or sustained-release formulation to an immediate-release formulation, and to monitor patients more frequently immediately following the surgery.
CASE CONTINUED
Related Resources
- Colvin C, Tsia W, Silverman AL, et al. Nothing up his sleeve: decompensation after bariatric surgery. Current Psychiatry. 2021;20(4):15-19. doi:10.12788/cp.010
Drug Brand Names
Alprazolam • Xanax
Amlodipine • Norvasc
Atorvastatin • Lipitor
Dulaglutide • Trulicity
Lisinopril • Zestril, Prinivil
Lithium • Eskalith, Lithobid
Metformin • Glucophage
Olanzapine • Zyprexa
Venlafaxine • Effexor
1. Obesity Treatments: Gastric Bypass Surgery. UCLA Health. Accessed April 4, 2021. http://surgery.ucla.edu/bariatrics-gastric-bypass
2. Thomas L. Gastric bypass more likely to require further treatment than gastric sleeve. News Medical. January 15, 2020. Accessed April 4, 2021. https://www.news-medical.net/news/20200115/Gastric-bypass-more-likely-to-require-further-treatment-than-gastric-sleeve.aspx
3. Lap Adjustable Gastric Banding. Laser Stone Surgery & Endoscopy Centre. September 5, 2016. Accessed April 4, 2021. http://www.laserstonesurgery.org/project/lap-adjustable-gastric-banding/
4. BPD/DS Weight-Loss Surgery. Johns Hopkins Medicine. Accessed April 4, 2021. https://www.hopkinsmedicine.org/health/treatment-tests-and-therapies/bpdds-weightloss-surgery
5. Holst JJ, Madsbad S, Bojsen-Møller KN, et al. Mechanisms in bariatric surgery: gut hormones, diabetes resolution, and weight loss. Surg Obes Relat Dis. 2018;14(5):708-714. doi:10.1016/j.soard.2018.03.003
6. Public Education Committee. Bariatric Surgery Procedures. American Society for Metabolic and Bariatric Surgery. Updated May 2021. Accessed September 4, 2021. https://asmbs.org/patients/bariatric-surgery-procedures
7. Padwal R, Brocks D, Sharma AM. A systematic review of drug absorption following bariatric surgery and its theoretical implications. Obes Rev. 2010;11(1):41-50. doi:10.1111/j.1467-789x.2009.00614.x
8. Godini L, Castellini G, Facchiano E, et al. Mood disorders and bariatric surgery patients: pre- and post- surgery clinical course- an overview. J Obes Weight Loss Medicat. 2016;2(1). doi:10.23937/2572-4010.1510012
9. Smith A, Henriksen B, Cohen A. Pharmacokinetic considerations in Roux-en-Y gastric bypass patients. Am J Health Syst Pharm. 2011;68(23):2241-2247. doi:10.2146/ajhp100630
10. Brocks DR, Ben-Eltriki M, Gabr RQ, et al. The effects of gastric bypass surgery on drug absorption and pharmacokinetics. Expert Opin Drug Metab Toxicol. 2012;8(12):1505-1519. doi:10.1517/17425255.2012.722757
11. Hamad GG, Helsel JC, Perel JM, et al. The effect of gastric bypass on the pharmacokinetics of serotonin reuptake inhibitors. Am J Psychiatry. 2012;169(3):256-263. doi:10.1176/appi.ajp.2011.11050719
12. Angeles PC, Robertsen I, Seeberg LT, et al. The influence of bariatric surgery on oral drug bioavailability in patients with obesity: a systematic review. Obes Rev. 2019;20(9):1299-1311. doi:10.1111/obr.12869
13. Laparoscopic Sleeve Gastrectomy. University of California San Francisco Department of Surgery. Accessed April 1, 2021. https://surgery.ucsf.edu/conditions--procedures/laparoscopic-sleeve-gastrectomy.aspx
14. Brethauer S, Schauer P. Laparoscopic Sleeve Gastrectomy: A Newcomer to Bariatric Surgery. Obesity Action Coalition. 2007. Accessed May 15, 2021. https://www.obesityaction.org/community/article-library/laparoscopic-sleeve-gastrectomy-a-newcomer-to-bariatric-surgery/
15. Roerig JL, Steffen K. Psychopharmacology and bariatric surgery. Eur Eat Disord Rev. 2015;23(6):463-469. doi:10.1002/erv.2396
16. Bland CM, Quidley AM, Love BL, et al. Long-term pharmacotherapy considerations in the bariatric surgery patient. A J Health Syst Pharm. 2016;73(16):1230-1242. doi:10.2146/ajhp151062
17. Lin YH, Liu SW, Wu HL, et al. Lithium toxicity with prolonged neurologic sequelae following sleeve gastrectomy: a case report and review of literature. Medicine (Baltimore). 2020;99(28):e21122. doi:10.1097/MD.0000000000021122
18. Lorico S, Colton B. Medication management and pharmacokinetic changes after bariatric surgery. Can Fam Physician. 2020;66(6):409-416.
19. Homan J, Schijns W, Aarts EO, et al. Treatment of vitamin and mineral deficiencies after biliopancreatic diversion with or without duodenal switch: a major challenge. Obes Surg. 2018;28(1):234-241. doi:10.1007/s11695-017-2841-0
20. Neovius M, Bruze G, Jacobson P, et al. Risk of suicide and non-fatal self-harm after bariatric surgery: results from two matched cohort studies. Lancet Diabetes Endocrinol. 2018;6(3):197-207. doi:10.1016/S2213-8587(17)30437-0
Ms. B, age 60, presents to the clinic with high blood pressure, hyperlipidemia, type 2 diabetes mellitus, depression, and anxiety. Her blood pressure is 138/82 mm Hg and pulse is 70 beats per minute. Her body mass index (BMI) is 41, which indicates she is obese. She has always struggled with her weight and has tried diet and lifestyle modifications, as well as medications, for the past 5 years with no success. Her current medication regimen includes lisinopril 40 mg daily, amlodipine 5 mg daily, atorvastatin 40 mg daily, metformin 500 mg twice daily, dulaglutide 0.75 mg weekly, lithium 600 mg daily, venlafaxine extended-release (XR) 150 mg daily, and alprazolam 0.5 mg as needed up to twice daily. Due to Ms. B’s BMI and because she has ≥1 comorbid health condition, her primary care physician refers her to a gastroenterologist to discuss gastric bypass surgery options.
Ms. B is scheduled for Roux-en-Y gastric bypass surgery. You need to determine if any changes should be made to her psychotropic medications after she undergoes this surgery.
There are multiple types of bariatric surgeries, including Roux-en-Y gastric bypass, sleeve gastrectomy, laparoscopic adjustable gastric band, and biliopancreatic diversion with duodenal switch (BPD/DS) (Figure1-4). These procedures all restrict the stomach’s capacity to hold food. In most cases, they also bypass areas of absorption in the intestine and cause increased secretion of hormones in the gut, including (but not limited to) peptide-YY (PYY) and glucagon-like peptide 1 (GLP-1). These hormonal changes impact several factors, including satiety, hunger, and blood sugar levels.5
Roux-en-Y is commonly referred to as the gold standard of weight loss surgery. It divides the top of the stomach into a smaller stomach pouch that connects directly to the small intestine to facilitate smaller meals and alters the release of gut hormones. Additionally, a segment of the small intestine that normally absorbs nutrients and medications is completely bypassed. In contrast, the sleeve gastrectomy removes approximately 80% of the stomach, consequently reducing the amount of food that can be consumed. The greatest impact of the sleeve gastrectomy procedure appears to result from changes in gut hormones. The adjustable gastric band procedure works by placing a band around the upper portion of the stomach to create a small pouch above the band to satisfy hunger with a smaller amount of food. Lastly, BPD/DS is a procedure that creates a tubular stomach pouch and bypasses a large portion of the small intestine. Like the gastric bypass and sleeve gastrectomy, BPD/DS affects gut hormones impacting hunger, satiety, and blood sugar control.
How bariatric surgery can affect drug absorption
As illustrated in the Table,6-19 each type of bariatric surgery may impact drug absorption differently depending on the mechanism by which the stomach is restricted.
Drug malabsorption is a concern for clinicians with patients who have undergone bariatric surgery. There is limited research measuring changes in psychotropic exposure and outcomes following bariatric surgery. A 2009 literature review by Padwal et al7 found that one-third of the 26 studies evaluated provided evidence of decreased absorption following bariatric surgery in patients taking medications that had intrinsic poor absorption, high lipophilicity, and/or undergo enterohepatic recirculation. In a review that included a small study of patients taking selective serotonin reuptake inhibitors or venlafaxine, Godini et al8 demonstrated that although there was a notable decrease in drug absorption closely following the surgery, drug absorption recovered for some patients 1 month after Roux-en-Y surgery. These reviews suggest patients who have undergone any form of bariatric surgery must be observed closely because drug absorption may vary based on the individual, the medication administered, and the amount of time postprocedure.
Until more research becomes available, current evidence supports recommendations to assist patients who have a decreased ability to absorb medications after gastric bypass surgery by switching from an extended-release formulation to an immediate-release or solution formulation. This allows patients to rely less on gastric mixing and unpredictable changes in drug release from extended- or controlled-release formulations.
Continue to: Aside from altered...
Aside from altered pharmacokinetics after bariatric surgery, many patients experience an increased risk of self-harm and suicide.20 Therefore, a continued emphasis on and reinforcement of proper antidepressant use and adjustment in these patients is important. This can be facilitated through frequent follow-up visits, either in-person or via telehealth.
Understanding the effect of bariatric surgery on drug absorption is critical to identifying a potential need to adjust a medication dose or formulation after the surgery. Available evidence and data suggest it is reasonable to switch from an extended- or sustained-release formulation to an immediate-release formulation, and to monitor patients more frequently immediately following the surgery.
CASE CONTINUED
Related Resources
- Colvin C, Tsia W, Silverman AL, et al. Nothing up his sleeve: decompensation after bariatric surgery. Current Psychiatry. 2021;20(4):15-19. doi:10.12788/cp.010
Drug Brand Names
Alprazolam • Xanax
Amlodipine • Norvasc
Atorvastatin • Lipitor
Dulaglutide • Trulicity
Lisinopril • Zestril, Prinivil
Lithium • Eskalith, Lithobid
Metformin • Glucophage
Olanzapine • Zyprexa
Venlafaxine • Effexor
Ms. B, age 60, presents to the clinic with high blood pressure, hyperlipidemia, type 2 diabetes mellitus, depression, and anxiety. Her blood pressure is 138/82 mm Hg and pulse is 70 beats per minute. Her body mass index (BMI) is 41, which indicates she is obese. She has always struggled with her weight and has tried diet and lifestyle modifications, as well as medications, for the past 5 years with no success. Her current medication regimen includes lisinopril 40 mg daily, amlodipine 5 mg daily, atorvastatin 40 mg daily, metformin 500 mg twice daily, dulaglutide 0.75 mg weekly, lithium 600 mg daily, venlafaxine extended-release (XR) 150 mg daily, and alprazolam 0.5 mg as needed up to twice daily. Due to Ms. B’s BMI and because she has ≥1 comorbid health condition, her primary care physician refers her to a gastroenterologist to discuss gastric bypass surgery options.
Ms. B is scheduled for Roux-en-Y gastric bypass surgery. You need to determine if any changes should be made to her psychotropic medications after she undergoes this surgery.
There are multiple types of bariatric surgeries, including Roux-en-Y gastric bypass, sleeve gastrectomy, laparoscopic adjustable gastric band, and biliopancreatic diversion with duodenal switch (BPD/DS) (Figure1-4). These procedures all restrict the stomach’s capacity to hold food. In most cases, they also bypass areas of absorption in the intestine and cause increased secretion of hormones in the gut, including (but not limited to) peptide-YY (PYY) and glucagon-like peptide 1 (GLP-1). These hormonal changes impact several factors, including satiety, hunger, and blood sugar levels.5
Roux-en-Y is commonly referred to as the gold standard of weight loss surgery. It divides the top of the stomach into a smaller stomach pouch that connects directly to the small intestine to facilitate smaller meals and alters the release of gut hormones. Additionally, a segment of the small intestine that normally absorbs nutrients and medications is completely bypassed. In contrast, the sleeve gastrectomy removes approximately 80% of the stomach, consequently reducing the amount of food that can be consumed. The greatest impact of the sleeve gastrectomy procedure appears to result from changes in gut hormones. The adjustable gastric band procedure works by placing a band around the upper portion of the stomach to create a small pouch above the band to satisfy hunger with a smaller amount of food. Lastly, BPD/DS is a procedure that creates a tubular stomach pouch and bypasses a large portion of the small intestine. Like the gastric bypass and sleeve gastrectomy, BPD/DS affects gut hormones impacting hunger, satiety, and blood sugar control.
How bariatric surgery can affect drug absorption
As illustrated in the Table,6-19 each type of bariatric surgery may impact drug absorption differently depending on the mechanism by which the stomach is restricted.
Drug malabsorption is a concern for clinicians with patients who have undergone bariatric surgery. There is limited research measuring changes in psychotropic exposure and outcomes following bariatric surgery. A 2009 literature review by Padwal et al7 found that one-third of the 26 studies evaluated provided evidence of decreased absorption following bariatric surgery in patients taking medications that had intrinsic poor absorption, high lipophilicity, and/or undergo enterohepatic recirculation. In a review that included a small study of patients taking selective serotonin reuptake inhibitors or venlafaxine, Godini et al8 demonstrated that although there was a notable decrease in drug absorption closely following the surgery, drug absorption recovered for some patients 1 month after Roux-en-Y surgery. These reviews suggest patients who have undergone any form of bariatric surgery must be observed closely because drug absorption may vary based on the individual, the medication administered, and the amount of time postprocedure.
Until more research becomes available, current evidence supports recommendations to assist patients who have a decreased ability to absorb medications after gastric bypass surgery by switching from an extended-release formulation to an immediate-release or solution formulation. This allows patients to rely less on gastric mixing and unpredictable changes in drug release from extended- or controlled-release formulations.
Continue to: Aside from altered...
Aside from altered pharmacokinetics after bariatric surgery, many patients experience an increased risk of self-harm and suicide.20 Therefore, a continued emphasis on and reinforcement of proper antidepressant use and adjustment in these patients is important. This can be facilitated through frequent follow-up visits, either in-person or via telehealth.
Understanding the effect of bariatric surgery on drug absorption is critical to identifying a potential need to adjust a medication dose or formulation after the surgery. Available evidence and data suggest it is reasonable to switch from an extended- or sustained-release formulation to an immediate-release formulation, and to monitor patients more frequently immediately following the surgery.
CASE CONTINUED
Related Resources
- Colvin C, Tsia W, Silverman AL, et al. Nothing up his sleeve: decompensation after bariatric surgery. Current Psychiatry. 2021;20(4):15-19. doi:10.12788/cp.010
Drug Brand Names
Alprazolam • Xanax
Amlodipine • Norvasc
Atorvastatin • Lipitor
Dulaglutide • Trulicity
Lisinopril • Zestril, Prinivil
Lithium • Eskalith, Lithobid
Metformin • Glucophage
Olanzapine • Zyprexa
Venlafaxine • Effexor
1. Obesity Treatments: Gastric Bypass Surgery. UCLA Health. Accessed April 4, 2021. http://surgery.ucla.edu/bariatrics-gastric-bypass
2. Thomas L. Gastric bypass more likely to require further treatment than gastric sleeve. News Medical. January 15, 2020. Accessed April 4, 2021. https://www.news-medical.net/news/20200115/Gastric-bypass-more-likely-to-require-further-treatment-than-gastric-sleeve.aspx
3. Lap Adjustable Gastric Banding. Laser Stone Surgery & Endoscopy Centre. September 5, 2016. Accessed April 4, 2021. http://www.laserstonesurgery.org/project/lap-adjustable-gastric-banding/
4. BPD/DS Weight-Loss Surgery. Johns Hopkins Medicine. Accessed April 4, 2021. https://www.hopkinsmedicine.org/health/treatment-tests-and-therapies/bpdds-weightloss-surgery
5. Holst JJ, Madsbad S, Bojsen-Møller KN, et al. Mechanisms in bariatric surgery: gut hormones, diabetes resolution, and weight loss. Surg Obes Relat Dis. 2018;14(5):708-714. doi:10.1016/j.soard.2018.03.003
6. Public Education Committee. Bariatric Surgery Procedures. American Society for Metabolic and Bariatric Surgery. Updated May 2021. Accessed September 4, 2021. https://asmbs.org/patients/bariatric-surgery-procedures
7. Padwal R, Brocks D, Sharma AM. A systematic review of drug absorption following bariatric surgery and its theoretical implications. Obes Rev. 2010;11(1):41-50. doi:10.1111/j.1467-789x.2009.00614.x
8. Godini L, Castellini G, Facchiano E, et al. Mood disorders and bariatric surgery patients: pre- and post- surgery clinical course- an overview. J Obes Weight Loss Medicat. 2016;2(1). doi:10.23937/2572-4010.1510012
9. Smith A, Henriksen B, Cohen A. Pharmacokinetic considerations in Roux-en-Y gastric bypass patients. Am J Health Syst Pharm. 2011;68(23):2241-2247. doi:10.2146/ajhp100630
10. Brocks DR, Ben-Eltriki M, Gabr RQ, et al. The effects of gastric bypass surgery on drug absorption and pharmacokinetics. Expert Opin Drug Metab Toxicol. 2012;8(12):1505-1519. doi:10.1517/17425255.2012.722757
11. Hamad GG, Helsel JC, Perel JM, et al. The effect of gastric bypass on the pharmacokinetics of serotonin reuptake inhibitors. Am J Psychiatry. 2012;169(3):256-263. doi:10.1176/appi.ajp.2011.11050719
12. Angeles PC, Robertsen I, Seeberg LT, et al. The influence of bariatric surgery on oral drug bioavailability in patients with obesity: a systematic review. Obes Rev. 2019;20(9):1299-1311. doi:10.1111/obr.12869
13. Laparoscopic Sleeve Gastrectomy. University of California San Francisco Department of Surgery. Accessed April 1, 2021. https://surgery.ucsf.edu/conditions--procedures/laparoscopic-sleeve-gastrectomy.aspx
14. Brethauer S, Schauer P. Laparoscopic Sleeve Gastrectomy: A Newcomer to Bariatric Surgery. Obesity Action Coalition. 2007. Accessed May 15, 2021. https://www.obesityaction.org/community/article-library/laparoscopic-sleeve-gastrectomy-a-newcomer-to-bariatric-surgery/
15. Roerig JL, Steffen K. Psychopharmacology and bariatric surgery. Eur Eat Disord Rev. 2015;23(6):463-469. doi:10.1002/erv.2396
16. Bland CM, Quidley AM, Love BL, et al. Long-term pharmacotherapy considerations in the bariatric surgery patient. A J Health Syst Pharm. 2016;73(16):1230-1242. doi:10.2146/ajhp151062
17. Lin YH, Liu SW, Wu HL, et al. Lithium toxicity with prolonged neurologic sequelae following sleeve gastrectomy: a case report and review of literature. Medicine (Baltimore). 2020;99(28):e21122. doi:10.1097/MD.0000000000021122
18. Lorico S, Colton B. Medication management and pharmacokinetic changes after bariatric surgery. Can Fam Physician. 2020;66(6):409-416.
19. Homan J, Schijns W, Aarts EO, et al. Treatment of vitamin and mineral deficiencies after biliopancreatic diversion with or without duodenal switch: a major challenge. Obes Surg. 2018;28(1):234-241. doi:10.1007/s11695-017-2841-0
20. Neovius M, Bruze G, Jacobson P, et al. Risk of suicide and non-fatal self-harm after bariatric surgery: results from two matched cohort studies. Lancet Diabetes Endocrinol. 2018;6(3):197-207. doi:10.1016/S2213-8587(17)30437-0
1. Obesity Treatments: Gastric Bypass Surgery. UCLA Health. Accessed April 4, 2021. http://surgery.ucla.edu/bariatrics-gastric-bypass
2. Thomas L. Gastric bypass more likely to require further treatment than gastric sleeve. News Medical. January 15, 2020. Accessed April 4, 2021. https://www.news-medical.net/news/20200115/Gastric-bypass-more-likely-to-require-further-treatment-than-gastric-sleeve.aspx
3. Lap Adjustable Gastric Banding. Laser Stone Surgery & Endoscopy Centre. September 5, 2016. Accessed April 4, 2021. http://www.laserstonesurgery.org/project/lap-adjustable-gastric-banding/
4. BPD/DS Weight-Loss Surgery. Johns Hopkins Medicine. Accessed April 4, 2021. https://www.hopkinsmedicine.org/health/treatment-tests-and-therapies/bpdds-weightloss-surgery
5. Holst JJ, Madsbad S, Bojsen-Møller KN, et al. Mechanisms in bariatric surgery: gut hormones, diabetes resolution, and weight loss. Surg Obes Relat Dis. 2018;14(5):708-714. doi:10.1016/j.soard.2018.03.003
6. Public Education Committee. Bariatric Surgery Procedures. American Society for Metabolic and Bariatric Surgery. Updated May 2021. Accessed September 4, 2021. https://asmbs.org/patients/bariatric-surgery-procedures
7. Padwal R, Brocks D, Sharma AM. A systematic review of drug absorption following bariatric surgery and its theoretical implications. Obes Rev. 2010;11(1):41-50. doi:10.1111/j.1467-789x.2009.00614.x
8. Godini L, Castellini G, Facchiano E, et al. Mood disorders and bariatric surgery patients: pre- and post- surgery clinical course- an overview. J Obes Weight Loss Medicat. 2016;2(1). doi:10.23937/2572-4010.1510012
9. Smith A, Henriksen B, Cohen A. Pharmacokinetic considerations in Roux-en-Y gastric bypass patients. Am J Health Syst Pharm. 2011;68(23):2241-2247. doi:10.2146/ajhp100630
10. Brocks DR, Ben-Eltriki M, Gabr RQ, et al. The effects of gastric bypass surgery on drug absorption and pharmacokinetics. Expert Opin Drug Metab Toxicol. 2012;8(12):1505-1519. doi:10.1517/17425255.2012.722757
11. Hamad GG, Helsel JC, Perel JM, et al. The effect of gastric bypass on the pharmacokinetics of serotonin reuptake inhibitors. Am J Psychiatry. 2012;169(3):256-263. doi:10.1176/appi.ajp.2011.11050719
12. Angeles PC, Robertsen I, Seeberg LT, et al. The influence of bariatric surgery on oral drug bioavailability in patients with obesity: a systematic review. Obes Rev. 2019;20(9):1299-1311. doi:10.1111/obr.12869
13. Laparoscopic Sleeve Gastrectomy. University of California San Francisco Department of Surgery. Accessed April 1, 2021. https://surgery.ucsf.edu/conditions--procedures/laparoscopic-sleeve-gastrectomy.aspx
14. Brethauer S, Schauer P. Laparoscopic Sleeve Gastrectomy: A Newcomer to Bariatric Surgery. Obesity Action Coalition. 2007. Accessed May 15, 2021. https://www.obesityaction.org/community/article-library/laparoscopic-sleeve-gastrectomy-a-newcomer-to-bariatric-surgery/
15. Roerig JL, Steffen K. Psychopharmacology and bariatric surgery. Eur Eat Disord Rev. 2015;23(6):463-469. doi:10.1002/erv.2396
16. Bland CM, Quidley AM, Love BL, et al. Long-term pharmacotherapy considerations in the bariatric surgery patient. A J Health Syst Pharm. 2016;73(16):1230-1242. doi:10.2146/ajhp151062
17. Lin YH, Liu SW, Wu HL, et al. Lithium toxicity with prolonged neurologic sequelae following sleeve gastrectomy: a case report and review of literature. Medicine (Baltimore). 2020;99(28):e21122. doi:10.1097/MD.0000000000021122
18. Lorico S, Colton B. Medication management and pharmacokinetic changes after bariatric surgery. Can Fam Physician. 2020;66(6):409-416.
19. Homan J, Schijns W, Aarts EO, et al. Treatment of vitamin and mineral deficiencies after biliopancreatic diversion with or without duodenal switch: a major challenge. Obes Surg. 2018;28(1):234-241. doi:10.1007/s11695-017-2841-0
20. Neovius M, Bruze G, Jacobson P, et al. Risk of suicide and non-fatal self-harm after bariatric surgery: results from two matched cohort studies. Lancet Diabetes Endocrinol. 2018;6(3):197-207. doi:10.1016/S2213-8587(17)30437-0
Impaired cognition in a patient with schizophrenia and HIV
CASE Psychotic episode in a patient with HIV
Mr. F, age 32, has schizophrenia and HIV. He presents to the emergency department with auditory and visual hallucinations in addition to paranoia. The treatment team refers him to the state psychiatric facility on an involuntary hold. Mr. F has had multiple previous hospitalizations, none of which had resulted in successful treatment. According to his most recent records, Mr. F failed to improve while taking olanzapine. Upon examination, Mr. F reports he hears command auditory hallucinations to hurt others and endorses paranoia. He is agitated, with a constricted affect, and his thought content is paranoid, disorganized, and circumstantial. Mr. F provides vague and evasive answers upon admission. His physical examination is unremarkable. He has an eighth-grade education level and limited insight into his illnesses. His Positive and Negative Syndrome Scale (PANSS) score is 122, indicating severe symptoms. The PANSS score is formulated based on 30 items, each scored between 1 and 7. Higher scores indicate more severe symptoms.
[polldaddy:11167946]
The authors’ observations
Compared to other medically ill patients, those with AIDS are 7 times more likely to experience EPS associated with antipsychotics. This may be a result of HIV infiltration of the basal ganglia causing regional changes that predispose these patients to EPS.
[polldaddy:11167948]
TREATMENT Haloperidol and antiretroviral therapy
The treatment team decides to start Mr. F on haloperidol for his psychotic symptoms as well as bictegravir, emtricitabine, and tenofovir for HIV. One week after admission, the team starts Mr. F on haloperidol decanoate 150 mg IM, and continues oral haloperidol and antiretroviral therapy. Mr. F reports some improvement in his hallucinations and appears to have reduced paranoia. He attends psychotherapy treatment groups over the next several days and scores 80 on a retrospective PANSS assessment (Figure 1). Mr. F receives haloperidol decanoate 200 mg IM 28 days after his first dose, and his oral haloperidol dose is reduced.
During the following 2 weeks, Mr. F endorses continued improvement of his symptoms and insight and begins discharge planning by calling his sister to discuss living arrangements. However, his mental state begins to decline; he becomes paranoid, withdrawn, and irritable, and endorses increased hallucinations. His PANSS score is 87, and he scores 11 on the Montreal Cognitive Assessment (MoCA), indicating moderate cognitive impairment. MoCA scores range from 0 to 30, with scores <10 indicating severe impairment, 10 to 17 indicating moderate impairment, 18 to 25 indicating mild impairment, and 26 to 30 considered normal. Figure 2 shows a timeline of Mr. F’s MoCA scores during treatment.
The treatment team increases the dose of haloperidol, and Mr. F continues to receive haloperidol deaconate injections monthly. After an adequate trial of haloperidol, the patient exhibits only partial response to treatment—his symptoms wax and wane—and he continues to display limited insight into both his mental illness and HIV diagnosis. Another PANSS assessment yields an essentially unchanged score of 88.
After a discussion of risks and benefits, Mr. F consents to initiating clozapine. The treatment team starts clozapine 25 mg/d and increases the dosage to 400 mg in the evening with a concomitant clozapine level of 487 ng/mL. Mr. F’s absolute neutrophil count was within normal limits (2,500 to 6,000 µL) during this period for weekly complete blood cell count monitoring. Over the next few weeks, his MoCA score increases to 17 and PANSS score decreases to 52. Haloperidol decanoate 200 mg IM is discontinued 3 days after Mr. F received a dose of clozapine 400 mg at bedtime. After an additional 2 weeks of clozapine at the same dosage, Mr. F scores 20 on the MoCA, an increase of 9 points from his baseline score while receiving haloperidol. There is a washout period for haloperidol decanoate and oral haloperidol before he completes a third MoCA. Mr. F participates in a discussion regarding his HIV diagnosis and the importance of consistently continuing treatment for this chronic infection. After some education, he has a better understanding of his condition and is more insightful about wanting to remain compliant with clozapine and bictegravir, emtricitabine, and tenofovir for his HIV.
The authors’ observations
Many patients receive treatment for comorbid HIV and schizophrenia. Patients with schizophrenia and other psychoses are at increased risk of contracting HIV due to numerous psychosocial factors, including an increased frequency of illicit drug use as well as an increased propensity for high-risk sexual behaviors secondary to impaired neurocognitive functioning, delusions, and victimization.1 In addition to deficits in functioning related to psychiatric illness, patients with HIV also experience virus-related neurocognitive insults. After crossing the blood-brain barrier, HIV viral proteins circulate in the blood, inducing brain endothelial cells to release cytokines, causing neuroinflammation.2
Continue to: Recently, inflammation and inflammatory...
Recently, inflammation and inflammatory biomarkers have become an important topic of psychiatric research. A meta-analysis by Fraguas et al3 concluded that greater inflammation and oxidative stress might lead to poorer outcomes in patients with first-episode psychosis. Based on this evidence, inflammation associated with untreated HIV infection may compound the pre-existing neurocognitive decline seen in patients with schizophrenia and other psychoses, thereby contributing to poor outcomes and treatment-resistant pathology.
Clozapine has been the superior treatment for refractory and nonrefractory schizophrenia.4 Factor et al5 report there are limited basal ganglia reserves in patients with HIV, which make clozapine the preferred option due to its low potential for causing EPS.
In this case, starting Mr. F on clozapine and titrating to therapeutic blood levels was associated with improved MoCA scores. Low MoCA scores could be due to untreated HIV, as well as inadequately treated psychosis. For Mr. F, improved MoCA scores were associated with increased insight into his HIV. It is important to note that Mr. F’s improved MoCA score also coincided with discontinuing monthly haloperidol decanoate injections. Haloperidol and its metabolites are believed to cause some neurotoxicity at high doses, and can contribute to cognitive impairment. This may partially explain the increased MoCA score after Mr. F stopped receiving haloperidol decanoate monthly injections.6 For the first time, he felt the need to be on antiretroviral therapy for his HIV, and was able to understand the chronic nature of HIV infection.
The benefit of clozapine treatment for patients with schizophrenia and comorbid HIV extends beyond symptomatic control. Long-term and consistent treatment of schizophrenia can be a stepping stone for improving many psychosocial factors. Improved insight allows patients to better understand their illness, treatment regimen, and follow-up needs. Improved self-care contributes to increased adherence to treatment regimens and overall health.
It is likely that patients who are consistently treated for schizophrenia will also have an increased capacity to understand their HIV diagnosis. With gained understanding, patients may be more likely to adhere to highly active antiretroviral therapy (HAART) for HIV and attend follow-up appointments with infectious disease or primary care physicians. Furthermore, with adherence to HAART therapy, patients can enjoy improved quality and duration of life by raising CD4 counts and preventing progression to AIDS and AIDS-related infections.
Continue to: In the case of...
In the case of Mr. F, we noted significant improvement in MoCA scores following treatment with clozapine. This led to improved insight into understanding the chronicity of HIV, understanding the complications of not being treated, and adherence to HAART medication. Improved cognition, as evidenced by an increased MoCA score, can significantly improve patient insight and adherence with medication.7 Insight into illness is particularly important when managing a patient with a chronic infectious illness such as HIV, where consistency with the medication regimen can decrease mortality and improve quality of life.8 Furthermore, with close monitoring, clozapine was a safe treatment option for this patient with HIV and schizophrenia.
Bottom Line
Patients with schizophrenia are at an increased risk of contracting HIV, and untreated schizophrenia decreases the likelihood patients will adhere to highly active antiretroviral therapy (HAART). Clozapine treatment in comorbid HIV and schizophrenia can improve cognition and insight into HIV diagnosis, possibly increasing the likelihood patients will remain compliant with HAART.
Related Resources
- Diduch MN, Campbell RH, Borovicka M, et al. Treating psychosis in patients with HIV/AIDS. Current Psychiatry. 2018;17(5):35-36,41-44,46.
Drug Brand Names
Bictegravir, emtricitabine, and tenofovir • Biktarvy
Clozapine • Clozaril
Haloperidol • Haldol
Haloperidol decanoate • Haldol decanoate
Olanzapine • Zyprexa
Ziprasidone • Geodon
1. Bahorik AL, Newhill CE, Eack SM. Neurocognitive functioning of individuals with schizophrenia: using and not using drugs. Schizophrenia Bull. 2014;40(4):856-867. doi:10.1093/schbul/sbt099
2. Hong S, Banks WA. Role of the immune system in HIV-associated neuroinflammation and neurocognitive implications. Brain Behav Immun. 2015;45:1-12. doi:10.1016/j.bbi.2014.10.008
3. Fraguas D, Díaz-Caneja CM, Rodríguez-Quiroga A, et al. Oxidative stress and inflammation in early onset first episode psychosis: a systematic review and meta-analysis. Int J Neuropsychopharmacol. 2017;20(6):435-444. doi:10.1093/ijnp/pyx015
4. Wahlbeck K, Cheine M, Essali A, et al. Evidence of clozapine’s effectiveness in schizophrenia: a systematic review and meta-analysis of randomized trials. Am J Psychiatry. 1999;156(7):990-999.
5. Factor SA, Brown D, Molho ES, et al. Clozapine: a 2-year open trial in Parkinson’s disease patients with psychosis. Neurology. 1994;44(3 Pt 1):544-546.
6. Raudenska M, Gumulec J, Babula P, et al. Haloperidol cytotoxicity and its relation to oxidative stress. Mini Rev Med Chem. 2013;13(14):1993-1998. doi:10.2174/13895575113136660100
7. El Abdellati K, De Picker L, Morrens M. Antipsychotic treatment failure: a systematic review on risk factors and interventions for treatment adherence in psychosis. Front Neurosci. 2020;14:531763. doi:10.3389/fnins.2020.531763
8. Margalho R, Pereira M, Ouakinin S, et al. Adesão à HAART, qualidade de vida e sintomat ologia psicopat ológica em doentes infectados pelo VIH/SIDA [Adherence to HAART, quality of life and psychopathological symptoms among HIV/AIDS infected patients]. Acta Med Port. 2011;24 Suppl 2:539-548.
CASE Psychotic episode in a patient with HIV
Mr. F, age 32, has schizophrenia and HIV. He presents to the emergency department with auditory and visual hallucinations in addition to paranoia. The treatment team refers him to the state psychiatric facility on an involuntary hold. Mr. F has had multiple previous hospitalizations, none of which had resulted in successful treatment. According to his most recent records, Mr. F failed to improve while taking olanzapine. Upon examination, Mr. F reports he hears command auditory hallucinations to hurt others and endorses paranoia. He is agitated, with a constricted affect, and his thought content is paranoid, disorganized, and circumstantial. Mr. F provides vague and evasive answers upon admission. His physical examination is unremarkable. He has an eighth-grade education level and limited insight into his illnesses. His Positive and Negative Syndrome Scale (PANSS) score is 122, indicating severe symptoms. The PANSS score is formulated based on 30 items, each scored between 1 and 7. Higher scores indicate more severe symptoms.
[polldaddy:11167946]
The authors’ observations
Compared to other medically ill patients, those with AIDS are 7 times more likely to experience EPS associated with antipsychotics. This may be a result of HIV infiltration of the basal ganglia causing regional changes that predispose these patients to EPS.
[polldaddy:11167948]
TREATMENT Haloperidol and antiretroviral therapy
The treatment team decides to start Mr. F on haloperidol for his psychotic symptoms as well as bictegravir, emtricitabine, and tenofovir for HIV. One week after admission, the team starts Mr. F on haloperidol decanoate 150 mg IM, and continues oral haloperidol and antiretroviral therapy. Mr. F reports some improvement in his hallucinations and appears to have reduced paranoia. He attends psychotherapy treatment groups over the next several days and scores 80 on a retrospective PANSS assessment (Figure 1). Mr. F receives haloperidol decanoate 200 mg IM 28 days after his first dose, and his oral haloperidol dose is reduced.
During the following 2 weeks, Mr. F endorses continued improvement of his symptoms and insight and begins discharge planning by calling his sister to discuss living arrangements. However, his mental state begins to decline; he becomes paranoid, withdrawn, and irritable, and endorses increased hallucinations. His PANSS score is 87, and he scores 11 on the Montreal Cognitive Assessment (MoCA), indicating moderate cognitive impairment. MoCA scores range from 0 to 30, with scores <10 indicating severe impairment, 10 to 17 indicating moderate impairment, 18 to 25 indicating mild impairment, and 26 to 30 considered normal. Figure 2 shows a timeline of Mr. F’s MoCA scores during treatment.
The treatment team increases the dose of haloperidol, and Mr. F continues to receive haloperidol deaconate injections monthly. After an adequate trial of haloperidol, the patient exhibits only partial response to treatment—his symptoms wax and wane—and he continues to display limited insight into both his mental illness and HIV diagnosis. Another PANSS assessment yields an essentially unchanged score of 88.
After a discussion of risks and benefits, Mr. F consents to initiating clozapine. The treatment team starts clozapine 25 mg/d and increases the dosage to 400 mg in the evening with a concomitant clozapine level of 487 ng/mL. Mr. F’s absolute neutrophil count was within normal limits (2,500 to 6,000 µL) during this period for weekly complete blood cell count monitoring. Over the next few weeks, his MoCA score increases to 17 and PANSS score decreases to 52. Haloperidol decanoate 200 mg IM is discontinued 3 days after Mr. F received a dose of clozapine 400 mg at bedtime. After an additional 2 weeks of clozapine at the same dosage, Mr. F scores 20 on the MoCA, an increase of 9 points from his baseline score while receiving haloperidol. There is a washout period for haloperidol decanoate and oral haloperidol before he completes a third MoCA. Mr. F participates in a discussion regarding his HIV diagnosis and the importance of consistently continuing treatment for this chronic infection. After some education, he has a better understanding of his condition and is more insightful about wanting to remain compliant with clozapine and bictegravir, emtricitabine, and tenofovir for his HIV.
The authors’ observations
Many patients receive treatment for comorbid HIV and schizophrenia. Patients with schizophrenia and other psychoses are at increased risk of contracting HIV due to numerous psychosocial factors, including an increased frequency of illicit drug use as well as an increased propensity for high-risk sexual behaviors secondary to impaired neurocognitive functioning, delusions, and victimization.1 In addition to deficits in functioning related to psychiatric illness, patients with HIV also experience virus-related neurocognitive insults. After crossing the blood-brain barrier, HIV viral proteins circulate in the blood, inducing brain endothelial cells to release cytokines, causing neuroinflammation.2
Continue to: Recently, inflammation and inflammatory...
Recently, inflammation and inflammatory biomarkers have become an important topic of psychiatric research. A meta-analysis by Fraguas et al3 concluded that greater inflammation and oxidative stress might lead to poorer outcomes in patients with first-episode psychosis. Based on this evidence, inflammation associated with untreated HIV infection may compound the pre-existing neurocognitive decline seen in patients with schizophrenia and other psychoses, thereby contributing to poor outcomes and treatment-resistant pathology.
Clozapine has been the superior treatment for refractory and nonrefractory schizophrenia.4 Factor et al5 report there are limited basal ganglia reserves in patients with HIV, which make clozapine the preferred option due to its low potential for causing EPS.
In this case, starting Mr. F on clozapine and titrating to therapeutic blood levels was associated with improved MoCA scores. Low MoCA scores could be due to untreated HIV, as well as inadequately treated psychosis. For Mr. F, improved MoCA scores were associated with increased insight into his HIV. It is important to note that Mr. F’s improved MoCA score also coincided with discontinuing monthly haloperidol decanoate injections. Haloperidol and its metabolites are believed to cause some neurotoxicity at high doses, and can contribute to cognitive impairment. This may partially explain the increased MoCA score after Mr. F stopped receiving haloperidol decanoate monthly injections.6 For the first time, he felt the need to be on antiretroviral therapy for his HIV, and was able to understand the chronic nature of HIV infection.
The benefit of clozapine treatment for patients with schizophrenia and comorbid HIV extends beyond symptomatic control. Long-term and consistent treatment of schizophrenia can be a stepping stone for improving many psychosocial factors. Improved insight allows patients to better understand their illness, treatment regimen, and follow-up needs. Improved self-care contributes to increased adherence to treatment regimens and overall health.
It is likely that patients who are consistently treated for schizophrenia will also have an increased capacity to understand their HIV diagnosis. With gained understanding, patients may be more likely to adhere to highly active antiretroviral therapy (HAART) for HIV and attend follow-up appointments with infectious disease or primary care physicians. Furthermore, with adherence to HAART therapy, patients can enjoy improved quality and duration of life by raising CD4 counts and preventing progression to AIDS and AIDS-related infections.
Continue to: In the case of...
In the case of Mr. F, we noted significant improvement in MoCA scores following treatment with clozapine. This led to improved insight into understanding the chronicity of HIV, understanding the complications of not being treated, and adherence to HAART medication. Improved cognition, as evidenced by an increased MoCA score, can significantly improve patient insight and adherence with medication.7 Insight into illness is particularly important when managing a patient with a chronic infectious illness such as HIV, where consistency with the medication regimen can decrease mortality and improve quality of life.8 Furthermore, with close monitoring, clozapine was a safe treatment option for this patient with HIV and schizophrenia.
Bottom Line
Patients with schizophrenia are at an increased risk of contracting HIV, and untreated schizophrenia decreases the likelihood patients will adhere to highly active antiretroviral therapy (HAART). Clozapine treatment in comorbid HIV and schizophrenia can improve cognition and insight into HIV diagnosis, possibly increasing the likelihood patients will remain compliant with HAART.
Related Resources
- Diduch MN, Campbell RH, Borovicka M, et al. Treating psychosis in patients with HIV/AIDS. Current Psychiatry. 2018;17(5):35-36,41-44,46.
Drug Brand Names
Bictegravir, emtricitabine, and tenofovir • Biktarvy
Clozapine • Clozaril
Haloperidol • Haldol
Haloperidol decanoate • Haldol decanoate
Olanzapine • Zyprexa
Ziprasidone • Geodon
CASE Psychotic episode in a patient with HIV
Mr. F, age 32, has schizophrenia and HIV. He presents to the emergency department with auditory and visual hallucinations in addition to paranoia. The treatment team refers him to the state psychiatric facility on an involuntary hold. Mr. F has had multiple previous hospitalizations, none of which had resulted in successful treatment. According to his most recent records, Mr. F failed to improve while taking olanzapine. Upon examination, Mr. F reports he hears command auditory hallucinations to hurt others and endorses paranoia. He is agitated, with a constricted affect, and his thought content is paranoid, disorganized, and circumstantial. Mr. F provides vague and evasive answers upon admission. His physical examination is unremarkable. He has an eighth-grade education level and limited insight into his illnesses. His Positive and Negative Syndrome Scale (PANSS) score is 122, indicating severe symptoms. The PANSS score is formulated based on 30 items, each scored between 1 and 7. Higher scores indicate more severe symptoms.
[polldaddy:11167946]
The authors’ observations
Compared to other medically ill patients, those with AIDS are 7 times more likely to experience EPS associated with antipsychotics. This may be a result of HIV infiltration of the basal ganglia causing regional changes that predispose these patients to EPS.
[polldaddy:11167948]
TREATMENT Haloperidol and antiretroviral therapy
The treatment team decides to start Mr. F on haloperidol for his psychotic symptoms as well as bictegravir, emtricitabine, and tenofovir for HIV. One week after admission, the team starts Mr. F on haloperidol decanoate 150 mg IM, and continues oral haloperidol and antiretroviral therapy. Mr. F reports some improvement in his hallucinations and appears to have reduced paranoia. He attends psychotherapy treatment groups over the next several days and scores 80 on a retrospective PANSS assessment (Figure 1). Mr. F receives haloperidol decanoate 200 mg IM 28 days after his first dose, and his oral haloperidol dose is reduced.
During the following 2 weeks, Mr. F endorses continued improvement of his symptoms and insight and begins discharge planning by calling his sister to discuss living arrangements. However, his mental state begins to decline; he becomes paranoid, withdrawn, and irritable, and endorses increased hallucinations. His PANSS score is 87, and he scores 11 on the Montreal Cognitive Assessment (MoCA), indicating moderate cognitive impairment. MoCA scores range from 0 to 30, with scores <10 indicating severe impairment, 10 to 17 indicating moderate impairment, 18 to 25 indicating mild impairment, and 26 to 30 considered normal. Figure 2 shows a timeline of Mr. F’s MoCA scores during treatment.
The treatment team increases the dose of haloperidol, and Mr. F continues to receive haloperidol deaconate injections monthly. After an adequate trial of haloperidol, the patient exhibits only partial response to treatment—his symptoms wax and wane—and he continues to display limited insight into both his mental illness and HIV diagnosis. Another PANSS assessment yields an essentially unchanged score of 88.
After a discussion of risks and benefits, Mr. F consents to initiating clozapine. The treatment team starts clozapine 25 mg/d and increases the dosage to 400 mg in the evening with a concomitant clozapine level of 487 ng/mL. Mr. F’s absolute neutrophil count was within normal limits (2,500 to 6,000 µL) during this period for weekly complete blood cell count monitoring. Over the next few weeks, his MoCA score increases to 17 and PANSS score decreases to 52. Haloperidol decanoate 200 mg IM is discontinued 3 days after Mr. F received a dose of clozapine 400 mg at bedtime. After an additional 2 weeks of clozapine at the same dosage, Mr. F scores 20 on the MoCA, an increase of 9 points from his baseline score while receiving haloperidol. There is a washout period for haloperidol decanoate and oral haloperidol before he completes a third MoCA. Mr. F participates in a discussion regarding his HIV diagnosis and the importance of consistently continuing treatment for this chronic infection. After some education, he has a better understanding of his condition and is more insightful about wanting to remain compliant with clozapine and bictegravir, emtricitabine, and tenofovir for his HIV.
The authors’ observations
Many patients receive treatment for comorbid HIV and schizophrenia. Patients with schizophrenia and other psychoses are at increased risk of contracting HIV due to numerous psychosocial factors, including an increased frequency of illicit drug use as well as an increased propensity for high-risk sexual behaviors secondary to impaired neurocognitive functioning, delusions, and victimization.1 In addition to deficits in functioning related to psychiatric illness, patients with HIV also experience virus-related neurocognitive insults. After crossing the blood-brain barrier, HIV viral proteins circulate in the blood, inducing brain endothelial cells to release cytokines, causing neuroinflammation.2
Continue to: Recently, inflammation and inflammatory...
Recently, inflammation and inflammatory biomarkers have become an important topic of psychiatric research. A meta-analysis by Fraguas et al3 concluded that greater inflammation and oxidative stress might lead to poorer outcomes in patients with first-episode psychosis. Based on this evidence, inflammation associated with untreated HIV infection may compound the pre-existing neurocognitive decline seen in patients with schizophrenia and other psychoses, thereby contributing to poor outcomes and treatment-resistant pathology.
Clozapine has been the superior treatment for refractory and nonrefractory schizophrenia.4 Factor et al5 report there are limited basal ganglia reserves in patients with HIV, which make clozapine the preferred option due to its low potential for causing EPS.
In this case, starting Mr. F on clozapine and titrating to therapeutic blood levels was associated with improved MoCA scores. Low MoCA scores could be due to untreated HIV, as well as inadequately treated psychosis. For Mr. F, improved MoCA scores were associated with increased insight into his HIV. It is important to note that Mr. F’s improved MoCA score also coincided with discontinuing monthly haloperidol decanoate injections. Haloperidol and its metabolites are believed to cause some neurotoxicity at high doses, and can contribute to cognitive impairment. This may partially explain the increased MoCA score after Mr. F stopped receiving haloperidol decanoate monthly injections.6 For the first time, he felt the need to be on antiretroviral therapy for his HIV, and was able to understand the chronic nature of HIV infection.
The benefit of clozapine treatment for patients with schizophrenia and comorbid HIV extends beyond symptomatic control. Long-term and consistent treatment of schizophrenia can be a stepping stone for improving many psychosocial factors. Improved insight allows patients to better understand their illness, treatment regimen, and follow-up needs. Improved self-care contributes to increased adherence to treatment regimens and overall health.
It is likely that patients who are consistently treated for schizophrenia will also have an increased capacity to understand their HIV diagnosis. With gained understanding, patients may be more likely to adhere to highly active antiretroviral therapy (HAART) for HIV and attend follow-up appointments with infectious disease or primary care physicians. Furthermore, with adherence to HAART therapy, patients can enjoy improved quality and duration of life by raising CD4 counts and preventing progression to AIDS and AIDS-related infections.
Continue to: In the case of...
In the case of Mr. F, we noted significant improvement in MoCA scores following treatment with clozapine. This led to improved insight into understanding the chronicity of HIV, understanding the complications of not being treated, and adherence to HAART medication. Improved cognition, as evidenced by an increased MoCA score, can significantly improve patient insight and adherence with medication.7 Insight into illness is particularly important when managing a patient with a chronic infectious illness such as HIV, where consistency with the medication regimen can decrease mortality and improve quality of life.8 Furthermore, with close monitoring, clozapine was a safe treatment option for this patient with HIV and schizophrenia.
Bottom Line
Patients with schizophrenia are at an increased risk of contracting HIV, and untreated schizophrenia decreases the likelihood patients will adhere to highly active antiretroviral therapy (HAART). Clozapine treatment in comorbid HIV and schizophrenia can improve cognition and insight into HIV diagnosis, possibly increasing the likelihood patients will remain compliant with HAART.
Related Resources
- Diduch MN, Campbell RH, Borovicka M, et al. Treating psychosis in patients with HIV/AIDS. Current Psychiatry. 2018;17(5):35-36,41-44,46.
Drug Brand Names
Bictegravir, emtricitabine, and tenofovir • Biktarvy
Clozapine • Clozaril
Haloperidol • Haldol
Haloperidol decanoate • Haldol decanoate
Olanzapine • Zyprexa
Ziprasidone • Geodon
1. Bahorik AL, Newhill CE, Eack SM. Neurocognitive functioning of individuals with schizophrenia: using and not using drugs. Schizophrenia Bull. 2014;40(4):856-867. doi:10.1093/schbul/sbt099
2. Hong S, Banks WA. Role of the immune system in HIV-associated neuroinflammation and neurocognitive implications. Brain Behav Immun. 2015;45:1-12. doi:10.1016/j.bbi.2014.10.008
3. Fraguas D, Díaz-Caneja CM, Rodríguez-Quiroga A, et al. Oxidative stress and inflammation in early onset first episode psychosis: a systematic review and meta-analysis. Int J Neuropsychopharmacol. 2017;20(6):435-444. doi:10.1093/ijnp/pyx015
4. Wahlbeck K, Cheine M, Essali A, et al. Evidence of clozapine’s effectiveness in schizophrenia: a systematic review and meta-analysis of randomized trials. Am J Psychiatry. 1999;156(7):990-999.
5. Factor SA, Brown D, Molho ES, et al. Clozapine: a 2-year open trial in Parkinson’s disease patients with psychosis. Neurology. 1994;44(3 Pt 1):544-546.
6. Raudenska M, Gumulec J, Babula P, et al. Haloperidol cytotoxicity and its relation to oxidative stress. Mini Rev Med Chem. 2013;13(14):1993-1998. doi:10.2174/13895575113136660100
7. El Abdellati K, De Picker L, Morrens M. Antipsychotic treatment failure: a systematic review on risk factors and interventions for treatment adherence in psychosis. Front Neurosci. 2020;14:531763. doi:10.3389/fnins.2020.531763
8. Margalho R, Pereira M, Ouakinin S, et al. Adesão à HAART, qualidade de vida e sintomat ologia psicopat ológica em doentes infectados pelo VIH/SIDA [Adherence to HAART, quality of life and psychopathological symptoms among HIV/AIDS infected patients]. Acta Med Port. 2011;24 Suppl 2:539-548.
1. Bahorik AL, Newhill CE, Eack SM. Neurocognitive functioning of individuals with schizophrenia: using and not using drugs. Schizophrenia Bull. 2014;40(4):856-867. doi:10.1093/schbul/sbt099
2. Hong S, Banks WA. Role of the immune system in HIV-associated neuroinflammation and neurocognitive implications. Brain Behav Immun. 2015;45:1-12. doi:10.1016/j.bbi.2014.10.008
3. Fraguas D, Díaz-Caneja CM, Rodríguez-Quiroga A, et al. Oxidative stress and inflammation in early onset first episode psychosis: a systematic review and meta-analysis. Int J Neuropsychopharmacol. 2017;20(6):435-444. doi:10.1093/ijnp/pyx015
4. Wahlbeck K, Cheine M, Essali A, et al. Evidence of clozapine’s effectiveness in schizophrenia: a systematic review and meta-analysis of randomized trials. Am J Psychiatry. 1999;156(7):990-999.
5. Factor SA, Brown D, Molho ES, et al. Clozapine: a 2-year open trial in Parkinson’s disease patients with psychosis. Neurology. 1994;44(3 Pt 1):544-546.
6. Raudenska M, Gumulec J, Babula P, et al. Haloperidol cytotoxicity and its relation to oxidative stress. Mini Rev Med Chem. 2013;13(14):1993-1998. doi:10.2174/13895575113136660100
7. El Abdellati K, De Picker L, Morrens M. Antipsychotic treatment failure: a systematic review on risk factors and interventions for treatment adherence in psychosis. Front Neurosci. 2020;14:531763. doi:10.3389/fnins.2020.531763
8. Margalho R, Pereira M, Ouakinin S, et al. Adesão à HAART, qualidade de vida e sintomat ologia psicopat ológica em doentes infectados pelo VIH/SIDA [Adherence to HAART, quality of life and psychopathological symptoms among HIV/AIDS infected patients]. Acta Med Port. 2011;24 Suppl 2:539-548.
Hope, help, and humor when facing a life-threatening illness
Editor’s note: Readers’ Forum is a department for correspondence from readers that is not in response to articles published in
My father, Morty Sosland, MD, was a psychiatrist in a community health setting when he was diagnosed with amyotrophic lateral sclerosis (ALS; Lou Gehrig’s disease) in April 2020. He continued to work until February 2021 and credits his ongoing resilience to what he refers to as “the 3 Hs”: hope, help, and humor. Although he can no longer speak, I was able to interview him over the advanced technology that is text messaging.
Sarah: Hi, Dad.
Morty: It’s Doctor Dad to you.
Sarah: I guess we are starting with humor, then?
Humor
Research has demonstrated that humor can have serious health benefits, such as decreasing stress-making hormones and altering dopamine activity.1 For individuals facing a life-threatening illness, humor can help them gain a sense of perspective in a situation that would otherwise feel overwhelming.
Sarah: I feel like a lot of the humor you used with patients was to help them gain perspective.
Continue to: Morty
Morty: Yes. I’d have to know the client well enough, though—and timing is important. My patients would come to me with a long list of challenges they had faced in the week, and I would say, “But besides that, everything’s good?”
Sarah: And besides the ALS, everything’s good?
Morty: Exactly. I’d also use magic or math tricks to make kids like coming to therapy or to reinforce important concepts.
Sarah: How has humor helped you cope?
Morty: Thinking about things in humorous ways has always been helpful. I used to say my Olympic sport was walking to the dining room with my walker. Unfortunately, I can’t do that anymore, so now my Olympic sport is getting out of bed. It’s a team sport.
Sarah: And that’s a good segue to…
Help
Countless studies have shown the impact of social support on health. Good social support can increase resilience, protect against mental illness, and even increase life expectancy.2 Support becomes even more critical when you are physically dependent on others due to illness.
Continue to: Sarah
Sarah: Was it difficult for you to accept help at first?
Morty: I would say yes—but at the same time, I accepted it because the illness was so shocking. I learned early on this was a fight that my family would also fight alongside me.
Sarah: I remember you would quote Fred Rogers.
Morty: Actually, it was Fred Rogers’ mother. She would tell her son during hard times, “Look for the helpers. You will always find people who are helping.” Helpers can be family members, friends, doctors, and aides, as well as others who have the same illness.
Hope
In the face of all life’s challenges, hope is important, but in the face of a life-threatening illness, hope must be multifaceted.3 In addition to hope for a cure, patients may focus their hopes on deepening relationships, maintaining dignity, or living each day to its fullest.
Morty: Early on in this illness, I chose to set a positive tone when I told people. I would say I have the top doctors and there is more research now than ever. Years ago, I wrote a children’s book with the mantra, “I say I can, I make a plan, I get right to it and then I do it.”4 My plan is to be around for at least 30 more years.
Sarah: Do you think it’s possible to hold acceptance and hope at the same time?
Morty: Acceptance and hope are not easy, but possible. I get down about this illness. In my dreams, I walk and talk, and most mornings I wake up and see my wheelchair and I think this is absurd or a different choice word. But I focus on the things I still can do, and that gives me a feeling of hope. I can read the latest research, I can enjoy moments of laughter, and I can spend time with my family and close friends.
1. Yim J. Therapeutic benefits of laughter in mental health: a theoretical review. Tohoku J Exp Med. 2016;239(3):243-249. doi:10.1620/tjem.239.243
2. Ozbay F, Johnson DC, Dimoulas E, et al. Social support and resilience to stress: from neurobiology to clinical practice. Psychiatry (Edgmont). 2007;4(5):35-40.
3. Hill DL, Feudnter C. Hope in the midst of terminal illness. In: Gallagher MW, Lopez SJ, eds. The Oxford Handbook of Hope. Oxford University Press; 2018:191-206.
4. Sosland MD. The Can Do Duck: A Story About Believing in Yourself. Can Do Duck Publishing; 2019.
Editor’s note: Readers’ Forum is a department for correspondence from readers that is not in response to articles published in
My father, Morty Sosland, MD, was a psychiatrist in a community health setting when he was diagnosed with amyotrophic lateral sclerosis (ALS; Lou Gehrig’s disease) in April 2020. He continued to work until February 2021 and credits his ongoing resilience to what he refers to as “the 3 Hs”: hope, help, and humor. Although he can no longer speak, I was able to interview him over the advanced technology that is text messaging.
Sarah: Hi, Dad.
Morty: It’s Doctor Dad to you.
Sarah: I guess we are starting with humor, then?
Humor
Research has demonstrated that humor can have serious health benefits, such as decreasing stress-making hormones and altering dopamine activity.1 For individuals facing a life-threatening illness, humor can help them gain a sense of perspective in a situation that would otherwise feel overwhelming.
Sarah: I feel like a lot of the humor you used with patients was to help them gain perspective.
Continue to: Morty
Morty: Yes. I’d have to know the client well enough, though—and timing is important. My patients would come to me with a long list of challenges they had faced in the week, and I would say, “But besides that, everything’s good?”
Sarah: And besides the ALS, everything’s good?
Morty: Exactly. I’d also use magic or math tricks to make kids like coming to therapy or to reinforce important concepts.
Sarah: How has humor helped you cope?
Morty: Thinking about things in humorous ways has always been helpful. I used to say my Olympic sport was walking to the dining room with my walker. Unfortunately, I can’t do that anymore, so now my Olympic sport is getting out of bed. It’s a team sport.
Sarah: And that’s a good segue to…
Help
Countless studies have shown the impact of social support on health. Good social support can increase resilience, protect against mental illness, and even increase life expectancy.2 Support becomes even more critical when you are physically dependent on others due to illness.
Continue to: Sarah
Sarah: Was it difficult for you to accept help at first?
Morty: I would say yes—but at the same time, I accepted it because the illness was so shocking. I learned early on this was a fight that my family would also fight alongside me.
Sarah: I remember you would quote Fred Rogers.
Morty: Actually, it was Fred Rogers’ mother. She would tell her son during hard times, “Look for the helpers. You will always find people who are helping.” Helpers can be family members, friends, doctors, and aides, as well as others who have the same illness.
Hope
In the face of all life’s challenges, hope is important, but in the face of a life-threatening illness, hope must be multifaceted.3 In addition to hope for a cure, patients may focus their hopes on deepening relationships, maintaining dignity, or living each day to its fullest.
Morty: Early on in this illness, I chose to set a positive tone when I told people. I would say I have the top doctors and there is more research now than ever. Years ago, I wrote a children’s book with the mantra, “I say I can, I make a plan, I get right to it and then I do it.”4 My plan is to be around for at least 30 more years.
Sarah: Do you think it’s possible to hold acceptance and hope at the same time?
Morty: Acceptance and hope are not easy, but possible. I get down about this illness. In my dreams, I walk and talk, and most mornings I wake up and see my wheelchair and I think this is absurd or a different choice word. But I focus on the things I still can do, and that gives me a feeling of hope. I can read the latest research, I can enjoy moments of laughter, and I can spend time with my family and close friends.
Editor’s note: Readers’ Forum is a department for correspondence from readers that is not in response to articles published in
My father, Morty Sosland, MD, was a psychiatrist in a community health setting when he was diagnosed with amyotrophic lateral sclerosis (ALS; Lou Gehrig’s disease) in April 2020. He continued to work until February 2021 and credits his ongoing resilience to what he refers to as “the 3 Hs”: hope, help, and humor. Although he can no longer speak, I was able to interview him over the advanced technology that is text messaging.
Sarah: Hi, Dad.
Morty: It’s Doctor Dad to you.
Sarah: I guess we are starting with humor, then?
Humor
Research has demonstrated that humor can have serious health benefits, such as decreasing stress-making hormones and altering dopamine activity.1 For individuals facing a life-threatening illness, humor can help them gain a sense of perspective in a situation that would otherwise feel overwhelming.
Sarah: I feel like a lot of the humor you used with patients was to help them gain perspective.
Continue to: Morty
Morty: Yes. I’d have to know the client well enough, though—and timing is important. My patients would come to me with a long list of challenges they had faced in the week, and I would say, “But besides that, everything’s good?”
Sarah: And besides the ALS, everything’s good?
Morty: Exactly. I’d also use magic or math tricks to make kids like coming to therapy or to reinforce important concepts.
Sarah: How has humor helped you cope?
Morty: Thinking about things in humorous ways has always been helpful. I used to say my Olympic sport was walking to the dining room with my walker. Unfortunately, I can’t do that anymore, so now my Olympic sport is getting out of bed. It’s a team sport.
Sarah: And that’s a good segue to…
Help
Countless studies have shown the impact of social support on health. Good social support can increase resilience, protect against mental illness, and even increase life expectancy.2 Support becomes even more critical when you are physically dependent on others due to illness.
Continue to: Sarah
Sarah: Was it difficult for you to accept help at first?
Morty: I would say yes—but at the same time, I accepted it because the illness was so shocking. I learned early on this was a fight that my family would also fight alongside me.
Sarah: I remember you would quote Fred Rogers.
Morty: Actually, it was Fred Rogers’ mother. She would tell her son during hard times, “Look for the helpers. You will always find people who are helping.” Helpers can be family members, friends, doctors, and aides, as well as others who have the same illness.
Hope
In the face of all life’s challenges, hope is important, but in the face of a life-threatening illness, hope must be multifaceted.3 In addition to hope for a cure, patients may focus their hopes on deepening relationships, maintaining dignity, or living each day to its fullest.
Morty: Early on in this illness, I chose to set a positive tone when I told people. I would say I have the top doctors and there is more research now than ever. Years ago, I wrote a children’s book with the mantra, “I say I can, I make a plan, I get right to it and then I do it.”4 My plan is to be around for at least 30 more years.
Sarah: Do you think it’s possible to hold acceptance and hope at the same time?
Morty: Acceptance and hope are not easy, but possible. I get down about this illness. In my dreams, I walk and talk, and most mornings I wake up and see my wheelchair and I think this is absurd or a different choice word. But I focus on the things I still can do, and that gives me a feeling of hope. I can read the latest research, I can enjoy moments of laughter, and I can spend time with my family and close friends.
1. Yim J. Therapeutic benefits of laughter in mental health: a theoretical review. Tohoku J Exp Med. 2016;239(3):243-249. doi:10.1620/tjem.239.243
2. Ozbay F, Johnson DC, Dimoulas E, et al. Social support and resilience to stress: from neurobiology to clinical practice. Psychiatry (Edgmont). 2007;4(5):35-40.
3. Hill DL, Feudnter C. Hope in the midst of terminal illness. In: Gallagher MW, Lopez SJ, eds. The Oxford Handbook of Hope. Oxford University Press; 2018:191-206.
4. Sosland MD. The Can Do Duck: A Story About Believing in Yourself. Can Do Duck Publishing; 2019.
1. Yim J. Therapeutic benefits of laughter in mental health: a theoretical review. Tohoku J Exp Med. 2016;239(3):243-249. doi:10.1620/tjem.239.243
2. Ozbay F, Johnson DC, Dimoulas E, et al. Social support and resilience to stress: from neurobiology to clinical practice. Psychiatry (Edgmont). 2007;4(5):35-40.
3. Hill DL, Feudnter C. Hope in the midst of terminal illness. In: Gallagher MW, Lopez SJ, eds. The Oxford Handbook of Hope. Oxford University Press; 2018:191-206.
4. Sosland MD. The Can Do Duck: A Story About Believing in Yourself. Can Do Duck Publishing; 2019.
Pregnancy termination: What psychiatrists need to know
Approximately half of pregnancies in the United States are unplanned, and approximately one-fifth of pregnancies end in elective termination.1 Psychiatrists who treat women of childbearing potential should understand critical aspects of abortion that could affect their patients’ mental health.
Discuss the potential for pregnancy with your patients. Individuals with psychiatric illness are less likely to adhere to contraceptive methods and are more likely to have unplanned pregnancies and detect pregnancies late.2 Women receiving psychiatric care could be at risk of not detecting pregnancy early enough to meet state laws that restrict the time frames in which abortions are allowed.
Understand that patients face barriers to abortion. Almost immediately after the Supreme Court overturned Roe v Wade in June 2022, abortion became illegal in several states. Even if abortion remains legal and available in your jurisdiction, patients could face barriers, including strict limits on abortion timing, monetary and travel challenges, preabortion counseling mandates, and timely access to an abortion provider.
Know that most patients can provide informed consent. Most patients with psychiatric illness have capacity to make medical decisions, including whether to consent to an abortion. Pro forma assessment is not necessary. Assessing capacity to consent to abortion should be the same as any other capacity assessment. If a woman lacks medical decision-making capacity, a substitute decision-maker must be used.
Recognize that ambivalence is normal. Even when a woman is certain about her decision to terminate a pregnancy, she might experience ambivalence. Ambivalence about important life decisions is common and should be validated and explored.3
Be aware of bias. As psychiatrists, we must ensure that our personal opinions about abortion do not impact patient care. An impartial and nondirective approach is key, and any effort to persuade or manipulate a woman’s decision is unethical. Because women with mental illness might be vulnerable to coercion, it is important to ensure that the woman’s choice is voluntary.
Accurately communicate information about mental health and abortion to your patients. Abortion does not worsen mental health. Research on abortion and mental health is rife with poorly designed studies that contain methodological flaws, including failure to control for confounding effects, such as pre-existing mental illness, and inadequate control group comparisons.4 For example, the correct comparison group in which to consider mental health outcomes for women who are seeking an abortion is those who sought an abortion but were not able to have one—not women with planned and desired pregnancies. The best predictor of postabortion mental wellness is preabortion mental health.5 Well-designed studies, such as the Turnaway Study, have demonstrated that abortion does not cause a significant increase in mental illness.6 The Turnaway Study was a well-designed, prospective study of thousands of women who obtained a wanted abortion. It compared many outcomes, including mental health, among women who wanted an abortion vs women who could not obtain a wanted abortion.
Continue to: Know that patients might not receive accurate information about the risks and impact of abortion
Know that patients might not receive accurate information about the risks and impact of abortion. A number of states have requirements—known as “informed consent laws”—that mandate physicians to provide state-authored informational packets about the risks and alternatives to abortion to patients seeking abortions. Some of this information is scientifically inaccurate, which poses a significant ethical dilemma for doctors who must choose between legal requirements and an obligation to scientific integrity.7
Recognize that abortion being illegal could negatively impact mental health. The consequences of being forced to carry out an unwanted pregnancy are profound. Women unable to obtain an abortion are more likely to have adverse health and pregnancy outcomes, live in poverty, stay with an abusive partner, and have difficulty bonding with the child.6 Abortion is highly stigmatized in the United States, and belonging to a stigmatized group is a risk factor for adverse mental health sequalae, including anxiety, depression, substance use, and cognitive deficits.4-6
Stay up-to-date on your state’s abortion laws. The legal landscape regarding abortion is changing rapidly, and it is important to stay abreast of these changes.
Restrictions on abortion likely will significantly affect women with psychiatric illness. As psychiatrists, we must be aware of the impact of the country’s changing laws will have on our patients and their mental health.
1. Guttmacher Institute. Accessed July 21, 2022. https://www.guttmacher.org/
2. Miller LJ. Sexuality, reproduction, and family planning in women with schizophrenia. Schizophr Bull. 1997;23(4):623-635. doi:10.1093/schbul/23.4.623
3. Brody BD, Chaudhry SK, Penzner JB, et al. A woman with major depression with psychotic features requesting a termination of pregnancy. Am J Psychiatry. 2016;173(1):12-15. doi:10.1176/appi.ajp.2015.15030380
4. Major B, Appelbaum M, Beckman L, et al. Abortion and mental health: Evaluating the evidence. Am Psychol. 2009;64(9):863-890. doi:10.1037/a0017497
5. Steinberg JR, Tschann JM, Furgerson D, et al. Psychosocial factors and pre-abortion psychological health: the significance of stigma. Soc Sci Med. 2016;150:67-75. doi:10.1016/j.socscimed.2015.12.007
6. ANSIRH. The Turnaway Study. Accessed June 29, 2022. https://www.ansirh.org/research/ongoing/turnaway-study
7. Daniels CR, Ferguson J, Howard G, et al. Informed or misinformed consent? Abortion policy in the United States. J Health Polit Policy Law. 2016;41(2):181-209. doi:10.1215/03616878-3476105
Approximately half of pregnancies in the United States are unplanned, and approximately one-fifth of pregnancies end in elective termination.1 Psychiatrists who treat women of childbearing potential should understand critical aspects of abortion that could affect their patients’ mental health.
Discuss the potential for pregnancy with your patients. Individuals with psychiatric illness are less likely to adhere to contraceptive methods and are more likely to have unplanned pregnancies and detect pregnancies late.2 Women receiving psychiatric care could be at risk of not detecting pregnancy early enough to meet state laws that restrict the time frames in which abortions are allowed.
Understand that patients face barriers to abortion. Almost immediately after the Supreme Court overturned Roe v Wade in June 2022, abortion became illegal in several states. Even if abortion remains legal and available in your jurisdiction, patients could face barriers, including strict limits on abortion timing, monetary and travel challenges, preabortion counseling mandates, and timely access to an abortion provider.
Know that most patients can provide informed consent. Most patients with psychiatric illness have capacity to make medical decisions, including whether to consent to an abortion. Pro forma assessment is not necessary. Assessing capacity to consent to abortion should be the same as any other capacity assessment. If a woman lacks medical decision-making capacity, a substitute decision-maker must be used.
Recognize that ambivalence is normal. Even when a woman is certain about her decision to terminate a pregnancy, she might experience ambivalence. Ambivalence about important life decisions is common and should be validated and explored.3
Be aware of bias. As psychiatrists, we must ensure that our personal opinions about abortion do not impact patient care. An impartial and nondirective approach is key, and any effort to persuade or manipulate a woman’s decision is unethical. Because women with mental illness might be vulnerable to coercion, it is important to ensure that the woman’s choice is voluntary.
Accurately communicate information about mental health and abortion to your patients. Abortion does not worsen mental health. Research on abortion and mental health is rife with poorly designed studies that contain methodological flaws, including failure to control for confounding effects, such as pre-existing mental illness, and inadequate control group comparisons.4 For example, the correct comparison group in which to consider mental health outcomes for women who are seeking an abortion is those who sought an abortion but were not able to have one—not women with planned and desired pregnancies. The best predictor of postabortion mental wellness is preabortion mental health.5 Well-designed studies, such as the Turnaway Study, have demonstrated that abortion does not cause a significant increase in mental illness.6 The Turnaway Study was a well-designed, prospective study of thousands of women who obtained a wanted abortion. It compared many outcomes, including mental health, among women who wanted an abortion vs women who could not obtain a wanted abortion.
Continue to: Know that patients might not receive accurate information about the risks and impact of abortion
Know that patients might not receive accurate information about the risks and impact of abortion. A number of states have requirements—known as “informed consent laws”—that mandate physicians to provide state-authored informational packets about the risks and alternatives to abortion to patients seeking abortions. Some of this information is scientifically inaccurate, which poses a significant ethical dilemma for doctors who must choose between legal requirements and an obligation to scientific integrity.7
Recognize that abortion being illegal could negatively impact mental health. The consequences of being forced to carry out an unwanted pregnancy are profound. Women unable to obtain an abortion are more likely to have adverse health and pregnancy outcomes, live in poverty, stay with an abusive partner, and have difficulty bonding with the child.6 Abortion is highly stigmatized in the United States, and belonging to a stigmatized group is a risk factor for adverse mental health sequalae, including anxiety, depression, substance use, and cognitive deficits.4-6
Stay up-to-date on your state’s abortion laws. The legal landscape regarding abortion is changing rapidly, and it is important to stay abreast of these changes.
Restrictions on abortion likely will significantly affect women with psychiatric illness. As psychiatrists, we must be aware of the impact of the country’s changing laws will have on our patients and their mental health.
Approximately half of pregnancies in the United States are unplanned, and approximately one-fifth of pregnancies end in elective termination.1 Psychiatrists who treat women of childbearing potential should understand critical aspects of abortion that could affect their patients’ mental health.
Discuss the potential for pregnancy with your patients. Individuals with psychiatric illness are less likely to adhere to contraceptive methods and are more likely to have unplanned pregnancies and detect pregnancies late.2 Women receiving psychiatric care could be at risk of not detecting pregnancy early enough to meet state laws that restrict the time frames in which abortions are allowed.
Understand that patients face barriers to abortion. Almost immediately after the Supreme Court overturned Roe v Wade in June 2022, abortion became illegal in several states. Even if abortion remains legal and available in your jurisdiction, patients could face barriers, including strict limits on abortion timing, monetary and travel challenges, preabortion counseling mandates, and timely access to an abortion provider.
Know that most patients can provide informed consent. Most patients with psychiatric illness have capacity to make medical decisions, including whether to consent to an abortion. Pro forma assessment is not necessary. Assessing capacity to consent to abortion should be the same as any other capacity assessment. If a woman lacks medical decision-making capacity, a substitute decision-maker must be used.
Recognize that ambivalence is normal. Even when a woman is certain about her decision to terminate a pregnancy, she might experience ambivalence. Ambivalence about important life decisions is common and should be validated and explored.3
Be aware of bias. As psychiatrists, we must ensure that our personal opinions about abortion do not impact patient care. An impartial and nondirective approach is key, and any effort to persuade or manipulate a woman’s decision is unethical. Because women with mental illness might be vulnerable to coercion, it is important to ensure that the woman’s choice is voluntary.
Accurately communicate information about mental health and abortion to your patients. Abortion does not worsen mental health. Research on abortion and mental health is rife with poorly designed studies that contain methodological flaws, including failure to control for confounding effects, such as pre-existing mental illness, and inadequate control group comparisons.4 For example, the correct comparison group in which to consider mental health outcomes for women who are seeking an abortion is those who sought an abortion but were not able to have one—not women with planned and desired pregnancies. The best predictor of postabortion mental wellness is preabortion mental health.5 Well-designed studies, such as the Turnaway Study, have demonstrated that abortion does not cause a significant increase in mental illness.6 The Turnaway Study was a well-designed, prospective study of thousands of women who obtained a wanted abortion. It compared many outcomes, including mental health, among women who wanted an abortion vs women who could not obtain a wanted abortion.
Continue to: Know that patients might not receive accurate information about the risks and impact of abortion
Know that patients might not receive accurate information about the risks and impact of abortion. A number of states have requirements—known as “informed consent laws”—that mandate physicians to provide state-authored informational packets about the risks and alternatives to abortion to patients seeking abortions. Some of this information is scientifically inaccurate, which poses a significant ethical dilemma for doctors who must choose between legal requirements and an obligation to scientific integrity.7
Recognize that abortion being illegal could negatively impact mental health. The consequences of being forced to carry out an unwanted pregnancy are profound. Women unable to obtain an abortion are more likely to have adverse health and pregnancy outcomes, live in poverty, stay with an abusive partner, and have difficulty bonding with the child.6 Abortion is highly stigmatized in the United States, and belonging to a stigmatized group is a risk factor for adverse mental health sequalae, including anxiety, depression, substance use, and cognitive deficits.4-6
Stay up-to-date on your state’s abortion laws. The legal landscape regarding abortion is changing rapidly, and it is important to stay abreast of these changes.
Restrictions on abortion likely will significantly affect women with psychiatric illness. As psychiatrists, we must be aware of the impact of the country’s changing laws will have on our patients and their mental health.
1. Guttmacher Institute. Accessed July 21, 2022. https://www.guttmacher.org/
2. Miller LJ. Sexuality, reproduction, and family planning in women with schizophrenia. Schizophr Bull. 1997;23(4):623-635. doi:10.1093/schbul/23.4.623
3. Brody BD, Chaudhry SK, Penzner JB, et al. A woman with major depression with psychotic features requesting a termination of pregnancy. Am J Psychiatry. 2016;173(1):12-15. doi:10.1176/appi.ajp.2015.15030380
4. Major B, Appelbaum M, Beckman L, et al. Abortion and mental health: Evaluating the evidence. Am Psychol. 2009;64(9):863-890. doi:10.1037/a0017497
5. Steinberg JR, Tschann JM, Furgerson D, et al. Psychosocial factors and pre-abortion psychological health: the significance of stigma. Soc Sci Med. 2016;150:67-75. doi:10.1016/j.socscimed.2015.12.007
6. ANSIRH. The Turnaway Study. Accessed June 29, 2022. https://www.ansirh.org/research/ongoing/turnaway-study
7. Daniels CR, Ferguson J, Howard G, et al. Informed or misinformed consent? Abortion policy in the United States. J Health Polit Policy Law. 2016;41(2):181-209. doi:10.1215/03616878-3476105
1. Guttmacher Institute. Accessed July 21, 2022. https://www.guttmacher.org/
2. Miller LJ. Sexuality, reproduction, and family planning in women with schizophrenia. Schizophr Bull. 1997;23(4):623-635. doi:10.1093/schbul/23.4.623
3. Brody BD, Chaudhry SK, Penzner JB, et al. A woman with major depression with psychotic features requesting a termination of pregnancy. Am J Psychiatry. 2016;173(1):12-15. doi:10.1176/appi.ajp.2015.15030380
4. Major B, Appelbaum M, Beckman L, et al. Abortion and mental health: Evaluating the evidence. Am Psychol. 2009;64(9):863-890. doi:10.1037/a0017497
5. Steinberg JR, Tschann JM, Furgerson D, et al. Psychosocial factors and pre-abortion psychological health: the significance of stigma. Soc Sci Med. 2016;150:67-75. doi:10.1016/j.socscimed.2015.12.007
6. ANSIRH. The Turnaway Study. Accessed June 29, 2022. https://www.ansirh.org/research/ongoing/turnaway-study
7. Daniels CR, Ferguson J, Howard G, et al. Informed or misinformed consent? Abortion policy in the United States. J Health Polit Policy Law. 2016;41(2):181-209. doi:10.1215/03616878-3476105
The impact of COVID-19 on adolescents’ mental health
While the COVID-19 pandemic has impacted the mental health of a wide range of individuals, its adverse effects have been particularly detrimental to adolescents. In this article, I discuss evidence that shows the effects of the pandemic on adolescent patients, potential reasons for this increased distress, and what types of coping mechanisms adolescents have used to counter these effects.
Increases in multiple measures of psychopathology
Multiple online surveys and other studies have documented the pandemic’s impact on younger individuals. In the United States, visits to emergency departments by pediatric patients increased in the months after the first lockdown period.1 Several studies found increased rates of anxiety and depression among adolescents during the COVID-19 pandemic.2,3 In an online survey of 359 children and 3,254 adolescents in China, 22% of respondents reported that they experienced depressive symptoms.3 In an online survey of 1,054 Canadian adolescents, 43% said they were “very concerned” about the pandemic.4 In an online survey of 7,353 adolescents in the United States, 37% reported suicidal ideation during the pandemic compared to 17% in 2017.5 A Chinese study found that smartphone and internet addiction was significantly associated with increased levels of depressive symptoms during the pandemic.3 In a survey in the Philippines, 16.3% of adolescents reported moderate-to-severe psychological impairment during the pandemic; the rates of COVID-19–related anxiety were higher among girls vs boys.6 Alcohol and cannabis use increased among Canadian adolescents during the pandemic, according to an online survey.7 Adolescents with anorexia nervosa reported a 70% increase in poor eating habits and more thoughts associated with eating disorders during the pandemic.8 A Danish study found that children and adolescents newly diagnosed with obsessive-compulsive disorder (OCD) or who had completed treatment exhibited worsening OCD, anxiety, and depressive symptoms during the pandemic.9 An online survey of 6,196 Chinese adolescents found that those with a higher number of pre-pandemic adverse childhood experiences, such as abuse and neglect, had elevated posttraumatic stress symptoms and anxiety during the onset of the pandemic.10
Underlying causes of pandemic-induced distress
Limited social connectedness during the pandemic is a major reason for distress among adolescents. A review of 80 studies found that social isolation and loneliness as a result of social distancing and quarantining were associated with an increased risk of depression, anxiety, suicidal ideation, and self-harm.11 Parents’ stress about the risks of COVID-19 was correlated with worsening mental health in their adolescent children.12 A Chinese study found that the amount of time students spent on smartphones and social media doubled during the pandemic.13 In an online survey of 7,890 Chinese adolescents, greater social media, internet, and smartphone use was associated with increased anxiety and depression.14 This may be in part the result of adolescents spending time reading COVID-related news.
Coping mechanisms to increase well-being
Researchers have identified several positive coping mechanisms adolescents employed during the pandemic. Although some data suggest that increased internet use raises the risk of COVID-related distress, for certain adolescents, using social media to stay connected with friends and relatives was a buffer for feelings of loneliness and might have increased mental well-being.15 Other common coping mechanisms include relying on faith, volunteering, and starting new hobbies.16 During the pandemic, there were higher rates of playing outside and increased physical activity, which correlated with positive mental health outcomes.16 An online survey of 1,040 adolescents found that those who looked to the future optimistically and confidently had a higher health-related quality of life.17
Continuing an emphasis on adolescent well-being
Although data are limited, adolescents can continue to use these coping mechanisms to maintain their well-being, even if COVID-related restrictions are lifted or reimplemented. During these difficult times, it is imperative for adolescents to get the mental health services they need, and for psychiatric clinicians to continue to find avenues to promote resilience and mental wellness among young patients.
1. Leeb RT, Bitsko RH, Radhakrishnan L, et al. Mental health–related emergency department visits among children aged <18 years during the COVID-19 pandemic—United States, January 1-October 17, 2020. MMWR Morb Mortal Wkly Rep. 2020;69(45):1675-1680. doi:10.15585/mmwr.mm6945a3
2. Oosterhoff B, Palmer CA, Wilson J, et al. Adolescents’ motivations to engage in social distancing during the COVID-19 pandemic: associations with mental and social health. J Adolesc Health. 2020;67(2):179-185. doi:10.1016/j.jadohealth.2020.05.004
3. Duan L, Shao X, Wang Y, et al. An investigation of mental health status of children and adolescents in China during the outbreak of COVID-19. J Affect Disord. 2020;275:112-118. doi:10.1016/j.jad.2020.06.029
4. Ellis WE, Dumas TM, Forbes LM. Physically isolated but socially connected: psychological adjustment and stress among adolescents during the initial COVID-19 crisis. Can J Behav Sci. 2020;52(3):177-187. doi:10.1037/cbs0000215
5. Murata S, Rezeppa T, Thoma B, et al. The psychiatric sequelae of the COVID-19 pandemic in adolescents, adults, and health care workers. Depress Anxiety. 2021;38(2):233-246. doi:10.1002/da.23120
6. Tee ML, Tee CA, Anlacan JP, et al. Psychological impact of COVID-19 pandemic in the Philippines. J Affect Disord. 2020;277:379-391. doi:10.1016/j.jad.2020.08.043
7. Dumas TM, Ellis W, Litt DM. What does adolescent substance use look like during the COVID-19 pandemic? Examining changes in frequency, social contexts, and pandemic-related predictors. J Adolesc Health. 2020;67(3):354-361. doi:10.1016/j.jadohealth.2020.06.018
8. Schlegl S, Maier J, Meule A, et al. Eating disorders in times of the COVID-19 pandemic—results from an online survey of patients with anorexia nervosa. Int J Eat Disord. 2020;53:1791-1800. doi:10.1002/eat.23374.
9. Nissen JB, Højgaard D, Thomsen PH. The immediate effect of COVID-19 pandemic on children and adolescents with obsessive compulsive disorder. BMC Psychiatry. 2020;20(1):511. doi:10.1186/s12888-020-02905-5
10. Guo J, Fu M, Liu D, et al. Is the psychological impact of exposure to COVID-19 stronger in adolescents with pre-pandemic maltreatment experiences? A survey of rural Chinese adolescents. Child Abuse Negl. 2020;110(Pt 2):104667. doi:10.1016/j.chiabu.2020.104667
11. Loades ME, Chatburn E, Higson-Sweeney N, et al. Rapid Systematic Review: The impact of social isolation and loneliness on the mental health of children and adolescents in the context of COVID-19. J Am Acad Child Adolesc Psychiatry. 2020;59(11):1218-1239.e3. doi:10.1016/j.jaac.2020.05.009
12. Spinelli M, Lionetti F, Setti A, et al. Parenting stress during the COVID-19 outbreak: socioeconomic and environmental risk factors and implications for children emotion regulation. Fam Process. 2021;60(2):639-653. doi:10.1111/famp.12601
13. Chen IH, Chen CY, Pakpour AH, et al. Internet-related behaviors and psychological distress among schoolchildren during COVID-19 school suspension. J Am Acad Child Adolesc Psychiatry. 2020;59(10):1099-1102.e1. doi:10.1016/j.jaac.2020.06.007
14. Li W, Zhang Y, Wang J, et al. Association of home quarantine and mental health among teenagers in Wuhan, China, during the COVID-19 pandemic. JAMA Pediatr. 2021;175(3):313-316. doi:10.1001/jamapediatrics.2020.5499
15. Janssen, LHC, Kullberg, MJ, Verkuil B, et al. Does the COVID-19 pandemic impact parents’ and adolescents’ well-being? An EMA-study on daily affect and parenting. PLoS One. 2020;15(10):e0240962. doi:10.1371/journal.pone.0240962
16. Banati P, Jones N, Youssef S. Intersecting vulnerabilities: the impacts of COVID-19 on the psycho-emotional lives of young people in low- and middle-income countries. Eur J Dev Res. 2020;32(5):1613-1638. doi:10.1057/s41287-020-00325-5
17. Ravens-Sieberer U, Kaman A, Otto C, et al. Mental health and quality of life in children and adolescents during the COVID-19 pandemic—results of the COPSY study. Dtsch Arztebl Int. 2020;117(48):828-829. doi:10.3238/arztebl.2020.0828
While the COVID-19 pandemic has impacted the mental health of a wide range of individuals, its adverse effects have been particularly detrimental to adolescents. In this article, I discuss evidence that shows the effects of the pandemic on adolescent patients, potential reasons for this increased distress, and what types of coping mechanisms adolescents have used to counter these effects.
Increases in multiple measures of psychopathology
Multiple online surveys and other studies have documented the pandemic’s impact on younger individuals. In the United States, visits to emergency departments by pediatric patients increased in the months after the first lockdown period.1 Several studies found increased rates of anxiety and depression among adolescents during the COVID-19 pandemic.2,3 In an online survey of 359 children and 3,254 adolescents in China, 22% of respondents reported that they experienced depressive symptoms.3 In an online survey of 1,054 Canadian adolescents, 43% said they were “very concerned” about the pandemic.4 In an online survey of 7,353 adolescents in the United States, 37% reported suicidal ideation during the pandemic compared to 17% in 2017.5 A Chinese study found that smartphone and internet addiction was significantly associated with increased levels of depressive symptoms during the pandemic.3 In a survey in the Philippines, 16.3% of adolescents reported moderate-to-severe psychological impairment during the pandemic; the rates of COVID-19–related anxiety were higher among girls vs boys.6 Alcohol and cannabis use increased among Canadian adolescents during the pandemic, according to an online survey.7 Adolescents with anorexia nervosa reported a 70% increase in poor eating habits and more thoughts associated with eating disorders during the pandemic.8 A Danish study found that children and adolescents newly diagnosed with obsessive-compulsive disorder (OCD) or who had completed treatment exhibited worsening OCD, anxiety, and depressive symptoms during the pandemic.9 An online survey of 6,196 Chinese adolescents found that those with a higher number of pre-pandemic adverse childhood experiences, such as abuse and neglect, had elevated posttraumatic stress symptoms and anxiety during the onset of the pandemic.10
Underlying causes of pandemic-induced distress
Limited social connectedness during the pandemic is a major reason for distress among adolescents. A review of 80 studies found that social isolation and loneliness as a result of social distancing and quarantining were associated with an increased risk of depression, anxiety, suicidal ideation, and self-harm.11 Parents’ stress about the risks of COVID-19 was correlated with worsening mental health in their adolescent children.12 A Chinese study found that the amount of time students spent on smartphones and social media doubled during the pandemic.13 In an online survey of 7,890 Chinese adolescents, greater social media, internet, and smartphone use was associated with increased anxiety and depression.14 This may be in part the result of adolescents spending time reading COVID-related news.
Coping mechanisms to increase well-being
Researchers have identified several positive coping mechanisms adolescents employed during the pandemic. Although some data suggest that increased internet use raises the risk of COVID-related distress, for certain adolescents, using social media to stay connected with friends and relatives was a buffer for feelings of loneliness and might have increased mental well-being.15 Other common coping mechanisms include relying on faith, volunteering, and starting new hobbies.16 During the pandemic, there were higher rates of playing outside and increased physical activity, which correlated with positive mental health outcomes.16 An online survey of 1,040 adolescents found that those who looked to the future optimistically and confidently had a higher health-related quality of life.17
Continuing an emphasis on adolescent well-being
Although data are limited, adolescents can continue to use these coping mechanisms to maintain their well-being, even if COVID-related restrictions are lifted or reimplemented. During these difficult times, it is imperative for adolescents to get the mental health services they need, and for psychiatric clinicians to continue to find avenues to promote resilience and mental wellness among young patients.
While the COVID-19 pandemic has impacted the mental health of a wide range of individuals, its adverse effects have been particularly detrimental to adolescents. In this article, I discuss evidence that shows the effects of the pandemic on adolescent patients, potential reasons for this increased distress, and what types of coping mechanisms adolescents have used to counter these effects.
Increases in multiple measures of psychopathology
Multiple online surveys and other studies have documented the pandemic’s impact on younger individuals. In the United States, visits to emergency departments by pediatric patients increased in the months after the first lockdown period.1 Several studies found increased rates of anxiety and depression among adolescents during the COVID-19 pandemic.2,3 In an online survey of 359 children and 3,254 adolescents in China, 22% of respondents reported that they experienced depressive symptoms.3 In an online survey of 1,054 Canadian adolescents, 43% said they were “very concerned” about the pandemic.4 In an online survey of 7,353 adolescents in the United States, 37% reported suicidal ideation during the pandemic compared to 17% in 2017.5 A Chinese study found that smartphone and internet addiction was significantly associated with increased levels of depressive symptoms during the pandemic.3 In a survey in the Philippines, 16.3% of adolescents reported moderate-to-severe psychological impairment during the pandemic; the rates of COVID-19–related anxiety were higher among girls vs boys.6 Alcohol and cannabis use increased among Canadian adolescents during the pandemic, according to an online survey.7 Adolescents with anorexia nervosa reported a 70% increase in poor eating habits and more thoughts associated with eating disorders during the pandemic.8 A Danish study found that children and adolescents newly diagnosed with obsessive-compulsive disorder (OCD) or who had completed treatment exhibited worsening OCD, anxiety, and depressive symptoms during the pandemic.9 An online survey of 6,196 Chinese adolescents found that those with a higher number of pre-pandemic adverse childhood experiences, such as abuse and neglect, had elevated posttraumatic stress symptoms and anxiety during the onset of the pandemic.10
Underlying causes of pandemic-induced distress
Limited social connectedness during the pandemic is a major reason for distress among adolescents. A review of 80 studies found that social isolation and loneliness as a result of social distancing and quarantining were associated with an increased risk of depression, anxiety, suicidal ideation, and self-harm.11 Parents’ stress about the risks of COVID-19 was correlated with worsening mental health in their adolescent children.12 A Chinese study found that the amount of time students spent on smartphones and social media doubled during the pandemic.13 In an online survey of 7,890 Chinese adolescents, greater social media, internet, and smartphone use was associated with increased anxiety and depression.14 This may be in part the result of adolescents spending time reading COVID-related news.
Coping mechanisms to increase well-being
Researchers have identified several positive coping mechanisms adolescents employed during the pandemic. Although some data suggest that increased internet use raises the risk of COVID-related distress, for certain adolescents, using social media to stay connected with friends and relatives was a buffer for feelings of loneliness and might have increased mental well-being.15 Other common coping mechanisms include relying on faith, volunteering, and starting new hobbies.16 During the pandemic, there were higher rates of playing outside and increased physical activity, which correlated with positive mental health outcomes.16 An online survey of 1,040 adolescents found that those who looked to the future optimistically and confidently had a higher health-related quality of life.17
Continuing an emphasis on adolescent well-being
Although data are limited, adolescents can continue to use these coping mechanisms to maintain their well-being, even if COVID-related restrictions are lifted or reimplemented. During these difficult times, it is imperative for adolescents to get the mental health services they need, and for psychiatric clinicians to continue to find avenues to promote resilience and mental wellness among young patients.
1. Leeb RT, Bitsko RH, Radhakrishnan L, et al. Mental health–related emergency department visits among children aged <18 years during the COVID-19 pandemic—United States, January 1-October 17, 2020. MMWR Morb Mortal Wkly Rep. 2020;69(45):1675-1680. doi:10.15585/mmwr.mm6945a3
2. Oosterhoff B, Palmer CA, Wilson J, et al. Adolescents’ motivations to engage in social distancing during the COVID-19 pandemic: associations with mental and social health. J Adolesc Health. 2020;67(2):179-185. doi:10.1016/j.jadohealth.2020.05.004
3. Duan L, Shao X, Wang Y, et al. An investigation of mental health status of children and adolescents in China during the outbreak of COVID-19. J Affect Disord. 2020;275:112-118. doi:10.1016/j.jad.2020.06.029
4. Ellis WE, Dumas TM, Forbes LM. Physically isolated but socially connected: psychological adjustment and stress among adolescents during the initial COVID-19 crisis. Can J Behav Sci. 2020;52(3):177-187. doi:10.1037/cbs0000215
5. Murata S, Rezeppa T, Thoma B, et al. The psychiatric sequelae of the COVID-19 pandemic in adolescents, adults, and health care workers. Depress Anxiety. 2021;38(2):233-246. doi:10.1002/da.23120
6. Tee ML, Tee CA, Anlacan JP, et al. Psychological impact of COVID-19 pandemic in the Philippines. J Affect Disord. 2020;277:379-391. doi:10.1016/j.jad.2020.08.043
7. Dumas TM, Ellis W, Litt DM. What does adolescent substance use look like during the COVID-19 pandemic? Examining changes in frequency, social contexts, and pandemic-related predictors. J Adolesc Health. 2020;67(3):354-361. doi:10.1016/j.jadohealth.2020.06.018
8. Schlegl S, Maier J, Meule A, et al. Eating disorders in times of the COVID-19 pandemic—results from an online survey of patients with anorexia nervosa. Int J Eat Disord. 2020;53:1791-1800. doi:10.1002/eat.23374.
9. Nissen JB, Højgaard D, Thomsen PH. The immediate effect of COVID-19 pandemic on children and adolescents with obsessive compulsive disorder. BMC Psychiatry. 2020;20(1):511. doi:10.1186/s12888-020-02905-5
10. Guo J, Fu M, Liu D, et al. Is the psychological impact of exposure to COVID-19 stronger in adolescents with pre-pandemic maltreatment experiences? A survey of rural Chinese adolescents. Child Abuse Negl. 2020;110(Pt 2):104667. doi:10.1016/j.chiabu.2020.104667
11. Loades ME, Chatburn E, Higson-Sweeney N, et al. Rapid Systematic Review: The impact of social isolation and loneliness on the mental health of children and adolescents in the context of COVID-19. J Am Acad Child Adolesc Psychiatry. 2020;59(11):1218-1239.e3. doi:10.1016/j.jaac.2020.05.009
12. Spinelli M, Lionetti F, Setti A, et al. Parenting stress during the COVID-19 outbreak: socioeconomic and environmental risk factors and implications for children emotion regulation. Fam Process. 2021;60(2):639-653. doi:10.1111/famp.12601
13. Chen IH, Chen CY, Pakpour AH, et al. Internet-related behaviors and psychological distress among schoolchildren during COVID-19 school suspension. J Am Acad Child Adolesc Psychiatry. 2020;59(10):1099-1102.e1. doi:10.1016/j.jaac.2020.06.007
14. Li W, Zhang Y, Wang J, et al. Association of home quarantine and mental health among teenagers in Wuhan, China, during the COVID-19 pandemic. JAMA Pediatr. 2021;175(3):313-316. doi:10.1001/jamapediatrics.2020.5499
15. Janssen, LHC, Kullberg, MJ, Verkuil B, et al. Does the COVID-19 pandemic impact parents’ and adolescents’ well-being? An EMA-study on daily affect and parenting. PLoS One. 2020;15(10):e0240962. doi:10.1371/journal.pone.0240962
16. Banati P, Jones N, Youssef S. Intersecting vulnerabilities: the impacts of COVID-19 on the psycho-emotional lives of young people in low- and middle-income countries. Eur J Dev Res. 2020;32(5):1613-1638. doi:10.1057/s41287-020-00325-5
17. Ravens-Sieberer U, Kaman A, Otto C, et al. Mental health and quality of life in children and adolescents during the COVID-19 pandemic—results of the COPSY study. Dtsch Arztebl Int. 2020;117(48):828-829. doi:10.3238/arztebl.2020.0828
1. Leeb RT, Bitsko RH, Radhakrishnan L, et al. Mental health–related emergency department visits among children aged <18 years during the COVID-19 pandemic—United States, January 1-October 17, 2020. MMWR Morb Mortal Wkly Rep. 2020;69(45):1675-1680. doi:10.15585/mmwr.mm6945a3
2. Oosterhoff B, Palmer CA, Wilson J, et al. Adolescents’ motivations to engage in social distancing during the COVID-19 pandemic: associations with mental and social health. J Adolesc Health. 2020;67(2):179-185. doi:10.1016/j.jadohealth.2020.05.004
3. Duan L, Shao X, Wang Y, et al. An investigation of mental health status of children and adolescents in China during the outbreak of COVID-19. J Affect Disord. 2020;275:112-118. doi:10.1016/j.jad.2020.06.029
4. Ellis WE, Dumas TM, Forbes LM. Physically isolated but socially connected: psychological adjustment and stress among adolescents during the initial COVID-19 crisis. Can J Behav Sci. 2020;52(3):177-187. doi:10.1037/cbs0000215
5. Murata S, Rezeppa T, Thoma B, et al. The psychiatric sequelae of the COVID-19 pandemic in adolescents, adults, and health care workers. Depress Anxiety. 2021;38(2):233-246. doi:10.1002/da.23120
6. Tee ML, Tee CA, Anlacan JP, et al. Psychological impact of COVID-19 pandemic in the Philippines. J Affect Disord. 2020;277:379-391. doi:10.1016/j.jad.2020.08.043
7. Dumas TM, Ellis W, Litt DM. What does adolescent substance use look like during the COVID-19 pandemic? Examining changes in frequency, social contexts, and pandemic-related predictors. J Adolesc Health. 2020;67(3):354-361. doi:10.1016/j.jadohealth.2020.06.018
8. Schlegl S, Maier J, Meule A, et al. Eating disorders in times of the COVID-19 pandemic—results from an online survey of patients with anorexia nervosa. Int J Eat Disord. 2020;53:1791-1800. doi:10.1002/eat.23374.
9. Nissen JB, Højgaard D, Thomsen PH. The immediate effect of COVID-19 pandemic on children and adolescents with obsessive compulsive disorder. BMC Psychiatry. 2020;20(1):511. doi:10.1186/s12888-020-02905-5
10. Guo J, Fu M, Liu D, et al. Is the psychological impact of exposure to COVID-19 stronger in adolescents with pre-pandemic maltreatment experiences? A survey of rural Chinese adolescents. Child Abuse Negl. 2020;110(Pt 2):104667. doi:10.1016/j.chiabu.2020.104667
11. Loades ME, Chatburn E, Higson-Sweeney N, et al. Rapid Systematic Review: The impact of social isolation and loneliness on the mental health of children and adolescents in the context of COVID-19. J Am Acad Child Adolesc Psychiatry. 2020;59(11):1218-1239.e3. doi:10.1016/j.jaac.2020.05.009
12. Spinelli M, Lionetti F, Setti A, et al. Parenting stress during the COVID-19 outbreak: socioeconomic and environmental risk factors and implications for children emotion regulation. Fam Process. 2021;60(2):639-653. doi:10.1111/famp.12601
13. Chen IH, Chen CY, Pakpour AH, et al. Internet-related behaviors and psychological distress among schoolchildren during COVID-19 school suspension. J Am Acad Child Adolesc Psychiatry. 2020;59(10):1099-1102.e1. doi:10.1016/j.jaac.2020.06.007
14. Li W, Zhang Y, Wang J, et al. Association of home quarantine and mental health among teenagers in Wuhan, China, during the COVID-19 pandemic. JAMA Pediatr. 2021;175(3):313-316. doi:10.1001/jamapediatrics.2020.5499
15. Janssen, LHC, Kullberg, MJ, Verkuil B, et al. Does the COVID-19 pandemic impact parents’ and adolescents’ well-being? An EMA-study on daily affect and parenting. PLoS One. 2020;15(10):e0240962. doi:10.1371/journal.pone.0240962
16. Banati P, Jones N, Youssef S. Intersecting vulnerabilities: the impacts of COVID-19 on the psycho-emotional lives of young people in low- and middle-income countries. Eur J Dev Res. 2020;32(5):1613-1638. doi:10.1057/s41287-020-00325-5
17. Ravens-Sieberer U, Kaman A, Otto C, et al. Mental health and quality of life in children and adolescents during the COVID-19 pandemic—results of the COPSY study. Dtsch Arztebl Int. 2020;117(48):828-829. doi:10.3238/arztebl.2020.0828
More on stigma
I just finished reading your editorial “A PSYCHIATRIC MANIFESTO: Stigma is hate speech and a hate crime” (
Our son went from an honor roll student before the pandemic to a child I barely recognized. Approximately 6 months into the pandemic, he was using drugs, vaping nicotine, destroying our property, and eloping at night. The journey of watching his decline and getting him help was agonizing. But the stigma around what was happening to him was an entirely separate animal.
Our society vilifies, ridicules, dismisses, and often makes fun of those with mental health issues. I experience it daily with my son and am on constant guard to shoot down any comments and to calmly teach those who say such cruel things. But the shame my son feels is the most devastating part. Although we keep reminding him that his condition is a medical condition like diabetes or heart disease, for a teenage boy, that makes no sense. He just wants to be “normal.” And living in a world that rarely represents mental illness this way, it’s almost a lost cause to get him to let go of this shame. All we can do is love him, be there for him, support him, and do what we can to educate those around us about the stigma of mental illness.
What a powerful and accurate article. Thank you for putting into words what I have been thinking and feeling, and for being as outraged as we are at how this vulnerable population is treated. My husband is a psychiatrist and we live in an affluent urban area, so we are not in the middle of nowhere with no knowledge of what is happening to our son. And despite that, we still suffer from the stigma.
Thank you, Dr. Nasrallah.
Name withheld
I need to take a moment to thank you for your editorial about stigma being hate speech and a hate crime. I really agree with you, and I think the way you formulated and articulated this message is very compelling.
I have focused on normalizing mental health differences among entrepreneurs as a destigmatization strategy (see https://www.sciencedirect.com/science/article/pii/S0883902622000027 and https://link.springer.com/article/10.1007/s11187-018-0059-8). Entrepreneurs clearly illustrate the fallacy of stigma. As a simple example, Elon Musk—the wealthiest person in the world—talks openly about being autistic, and possibly bipolar. These mental health differences help him create jobs and contribute to our shared prosperity. Nothing to be ashamed of there.
Thanks again for being such an effective advocate.
Michael A. Freeman, MD
Kentfield, California
Continue to: Thank you...
Thank you so much for your “Psychiatric Manifesto.” I will do my best to disseminate it amongst colleagues, patients, friends, family, and as many others as possible.
Daniel N. Pistone, MD
San Francisco, California
Once again, your words hit the pin on the head.
Robert W. Pollack, MD, ABPN, DLFAPA
Fort Myers, Florida
I just finished reading your editorial “A PSYCHIATRIC MANIFESTO: Stigma is hate speech and a hate crime” (
Our son went from an honor roll student before the pandemic to a child I barely recognized. Approximately 6 months into the pandemic, he was using drugs, vaping nicotine, destroying our property, and eloping at night. The journey of watching his decline and getting him help was agonizing. But the stigma around what was happening to him was an entirely separate animal.
Our society vilifies, ridicules, dismisses, and often makes fun of those with mental health issues. I experience it daily with my son and am on constant guard to shoot down any comments and to calmly teach those who say such cruel things. But the shame my son feels is the most devastating part. Although we keep reminding him that his condition is a medical condition like diabetes or heart disease, for a teenage boy, that makes no sense. He just wants to be “normal.” And living in a world that rarely represents mental illness this way, it’s almost a lost cause to get him to let go of this shame. All we can do is love him, be there for him, support him, and do what we can to educate those around us about the stigma of mental illness.
What a powerful and accurate article. Thank you for putting into words what I have been thinking and feeling, and for being as outraged as we are at how this vulnerable population is treated. My husband is a psychiatrist and we live in an affluent urban area, so we are not in the middle of nowhere with no knowledge of what is happening to our son. And despite that, we still suffer from the stigma.
Thank you, Dr. Nasrallah.
Name withheld
I need to take a moment to thank you for your editorial about stigma being hate speech and a hate crime. I really agree with you, and I think the way you formulated and articulated this message is very compelling.
I have focused on normalizing mental health differences among entrepreneurs as a destigmatization strategy (see https://www.sciencedirect.com/science/article/pii/S0883902622000027 and https://link.springer.com/article/10.1007/s11187-018-0059-8). Entrepreneurs clearly illustrate the fallacy of stigma. As a simple example, Elon Musk—the wealthiest person in the world—talks openly about being autistic, and possibly bipolar. These mental health differences help him create jobs and contribute to our shared prosperity. Nothing to be ashamed of there.
Thanks again for being such an effective advocate.
Michael A. Freeman, MD
Kentfield, California
Continue to: Thank you...
Thank you so much for your “Psychiatric Manifesto.” I will do my best to disseminate it amongst colleagues, patients, friends, family, and as many others as possible.
Daniel N. Pistone, MD
San Francisco, California
Once again, your words hit the pin on the head.
Robert W. Pollack, MD, ABPN, DLFAPA
Fort Myers, Florida
I just finished reading your editorial “A PSYCHIATRIC MANIFESTO: Stigma is hate speech and a hate crime” (
Our son went from an honor roll student before the pandemic to a child I barely recognized. Approximately 6 months into the pandemic, he was using drugs, vaping nicotine, destroying our property, and eloping at night. The journey of watching his decline and getting him help was agonizing. But the stigma around what was happening to him was an entirely separate animal.
Our society vilifies, ridicules, dismisses, and often makes fun of those with mental health issues. I experience it daily with my son and am on constant guard to shoot down any comments and to calmly teach those who say such cruel things. But the shame my son feels is the most devastating part. Although we keep reminding him that his condition is a medical condition like diabetes or heart disease, for a teenage boy, that makes no sense. He just wants to be “normal.” And living in a world that rarely represents mental illness this way, it’s almost a lost cause to get him to let go of this shame. All we can do is love him, be there for him, support him, and do what we can to educate those around us about the stigma of mental illness.
What a powerful and accurate article. Thank you for putting into words what I have been thinking and feeling, and for being as outraged as we are at how this vulnerable population is treated. My husband is a psychiatrist and we live in an affluent urban area, so we are not in the middle of nowhere with no knowledge of what is happening to our son. And despite that, we still suffer from the stigma.
Thank you, Dr. Nasrallah.
Name withheld
I need to take a moment to thank you for your editorial about stigma being hate speech and a hate crime. I really agree with you, and I think the way you formulated and articulated this message is very compelling.
I have focused on normalizing mental health differences among entrepreneurs as a destigmatization strategy (see https://www.sciencedirect.com/science/article/pii/S0883902622000027 and https://link.springer.com/article/10.1007/s11187-018-0059-8). Entrepreneurs clearly illustrate the fallacy of stigma. As a simple example, Elon Musk—the wealthiest person in the world—talks openly about being autistic, and possibly bipolar. These mental health differences help him create jobs and contribute to our shared prosperity. Nothing to be ashamed of there.
Thanks again for being such an effective advocate.
Michael A. Freeman, MD
Kentfield, California
Continue to: Thank you...
Thank you so much for your “Psychiatric Manifesto.” I will do my best to disseminate it amongst colleagues, patients, friends, family, and as many others as possible.
Daniel N. Pistone, MD
San Francisco, California
Once again, your words hit the pin on the head.
Robert W. Pollack, MD, ABPN, DLFAPA
Fort Myers, Florida
Episodes of visual disturbance
On the basis of the history, examination, and investigations, retinal migraine was diagnosed according to the International Classification of Headache Disorders, third edition (1.2 migraine with aura; 1.2.4 retinal migraine). This classification system describes retinal migraine as a subtype of migraine with aura.
Retinal migraine (also called ophthalmic or ocular migraine) is relatively rare but is sometimes a cause of transient monocular blindness in young adults. It manifests as recurrent attacks of unilateral visual disturbance (positive symptoms) or blindness (negative symptoms) lasting from minutes to 1 hour, associated with minimal or no headache.
Some patients describe a positive visual symptom/disturbance in a mosaic pattern of scotomata that gradually enlarge, producing total or near-total unilateral visual loss. Precipitating factors may include emotional stress, hypertension, and hormonal contraceptive pills, as well as exercise, high altitude, dehydration, smoking, hypoglycemia, and hyperthermia.
Retinal migraine is believed to result from transient vasospasm of the choroidal or retinal arteries. A history of recurrent attacks of transient monocular visual disturbance or blindness, with or without a headache and without other neurologic symptoms, can suggest retinal migraine. A personal or family history of migraine can confirm the diagnosis.
Ruling out eye disease or vascular causes, especially when risk factors for arteriosclerosis exist, is important; that is, the condition must be differentiated from ocular or vascular causes of transient monocular blindness, mainly carotid artery disease.
Carotid duplex ultrasonography, transcranial Doppler ultrasonography, magnetic resonance angiography, or CT angiography of the brain may be helpful. Fluorescein or cerebral angiography is rarely necessary. A hypercoagulability workup and evaluation of the erythrocyte sedimentation rate may be useful in excluding other coagulation disorders associated with retinal vasculopathy.
Regarding management, calcium-channel blockers have shown some efficacy. Even in patients with low blood pressure, nifedipine 10-20 mg/d is generally tolerated. From the available literature on treatment of this condition, it is recommended that triptans, ergots, and beta-blockers be used with caution or avoided in patients with retinal migraine owing to the potential for exacerbating vasoconstriction of the retinal artery. Transient vision loss in retinal migraine has been associated with future onset of permanent vision loss from occlusive conditions such as central retinal artery occlusion and branch retinal artery occlusion.
Jasmin Harpe, MD, MPH, Headache Fellow, Department of Neurology, Harvard University, John R. Graham Headache Center, Mass General Brigham, Boston, MA
Jasmin Harpe, MD, MPH, has disclosed no relevant financial relationships.
Image Quizzes are fictional or fictionalized clinical scenarios intended to provide evidence-based educational takeaways.
On the basis of the history, examination, and investigations, retinal migraine was diagnosed according to the International Classification of Headache Disorders, third edition (1.2 migraine with aura; 1.2.4 retinal migraine). This classification system describes retinal migraine as a subtype of migraine with aura.
Retinal migraine (also called ophthalmic or ocular migraine) is relatively rare but is sometimes a cause of transient monocular blindness in young adults. It manifests as recurrent attacks of unilateral visual disturbance (positive symptoms) or blindness (negative symptoms) lasting from minutes to 1 hour, associated with minimal or no headache.
Some patients describe a positive visual symptom/disturbance in a mosaic pattern of scotomata that gradually enlarge, producing total or near-total unilateral visual loss. Precipitating factors may include emotional stress, hypertension, and hormonal contraceptive pills, as well as exercise, high altitude, dehydration, smoking, hypoglycemia, and hyperthermia.
Retinal migraine is believed to result from transient vasospasm of the choroidal or retinal arteries. A history of recurrent attacks of transient monocular visual disturbance or blindness, with or without a headache and without other neurologic symptoms, can suggest retinal migraine. A personal or family history of migraine can confirm the diagnosis.
Ruling out eye disease or vascular causes, especially when risk factors for arteriosclerosis exist, is important; that is, the condition must be differentiated from ocular or vascular causes of transient monocular blindness, mainly carotid artery disease.
Carotid duplex ultrasonography, transcranial Doppler ultrasonography, magnetic resonance angiography, or CT angiography of the brain may be helpful. Fluorescein or cerebral angiography is rarely necessary. A hypercoagulability workup and evaluation of the erythrocyte sedimentation rate may be useful in excluding other coagulation disorders associated with retinal vasculopathy.
Regarding management, calcium-channel blockers have shown some efficacy. Even in patients with low blood pressure, nifedipine 10-20 mg/d is generally tolerated. From the available literature on treatment of this condition, it is recommended that triptans, ergots, and beta-blockers be used with caution or avoided in patients with retinal migraine owing to the potential for exacerbating vasoconstriction of the retinal artery. Transient vision loss in retinal migraine has been associated with future onset of permanent vision loss from occlusive conditions such as central retinal artery occlusion and branch retinal artery occlusion.
Jasmin Harpe, MD, MPH, Headache Fellow, Department of Neurology, Harvard University, John R. Graham Headache Center, Mass General Brigham, Boston, MA
Jasmin Harpe, MD, MPH, has disclosed no relevant financial relationships.
Image Quizzes are fictional or fictionalized clinical scenarios intended to provide evidence-based educational takeaways.
On the basis of the history, examination, and investigations, retinal migraine was diagnosed according to the International Classification of Headache Disorders, third edition (1.2 migraine with aura; 1.2.4 retinal migraine). This classification system describes retinal migraine as a subtype of migraine with aura.
Retinal migraine (also called ophthalmic or ocular migraine) is relatively rare but is sometimes a cause of transient monocular blindness in young adults. It manifests as recurrent attacks of unilateral visual disturbance (positive symptoms) or blindness (negative symptoms) lasting from minutes to 1 hour, associated with minimal or no headache.
Some patients describe a positive visual symptom/disturbance in a mosaic pattern of scotomata that gradually enlarge, producing total or near-total unilateral visual loss. Precipitating factors may include emotional stress, hypertension, and hormonal contraceptive pills, as well as exercise, high altitude, dehydration, smoking, hypoglycemia, and hyperthermia.
Retinal migraine is believed to result from transient vasospasm of the choroidal or retinal arteries. A history of recurrent attacks of transient monocular visual disturbance or blindness, with or without a headache and without other neurologic symptoms, can suggest retinal migraine. A personal or family history of migraine can confirm the diagnosis.
Ruling out eye disease or vascular causes, especially when risk factors for arteriosclerosis exist, is important; that is, the condition must be differentiated from ocular or vascular causes of transient monocular blindness, mainly carotid artery disease.
Carotid duplex ultrasonography, transcranial Doppler ultrasonography, magnetic resonance angiography, or CT angiography of the brain may be helpful. Fluorescein or cerebral angiography is rarely necessary. A hypercoagulability workup and evaluation of the erythrocyte sedimentation rate may be useful in excluding other coagulation disorders associated with retinal vasculopathy.
Regarding management, calcium-channel blockers have shown some efficacy. Even in patients with low blood pressure, nifedipine 10-20 mg/d is generally tolerated. From the available literature on treatment of this condition, it is recommended that triptans, ergots, and beta-blockers be used with caution or avoided in patients with retinal migraine owing to the potential for exacerbating vasoconstriction of the retinal artery. Transient vision loss in retinal migraine has been associated with future onset of permanent vision loss from occlusive conditions such as central retinal artery occlusion and branch retinal artery occlusion.
Jasmin Harpe, MD, MPH, Headache Fellow, Department of Neurology, Harvard University, John R. Graham Headache Center, Mass General Brigham, Boston, MA
Jasmin Harpe, MD, MPH, has disclosed no relevant financial relationships.
Image Quizzes are fictional or fictionalized clinical scenarios intended to provide evidence-based educational takeaways.
A 23-year-old woman presents with sudden recurrent episodes of visual disturbance (extreme blurriness and partial blindness) in her right eye. She had seven or eight episodes over 30 hours; each episode lasted for 5-7 minutes, with spontaneous and full recovery. These were not associated with flashes of light, tingling, numbness, fever, or headache. She was asymptomatic between episodes.
She had normal vision in her left eye during these episodes, which she checked by covering both eyes alternately with her hands. The only significant history was four episodes of migraine with aura 3 years ago, which resolved spontaneously and did not recur. Family history was noncontributory. She had no history of illicit drug use or alcohol use.
On examination, her vital signs were normal. Blood pressure was 110/80 mm Hg, pulse 85 beats/min, and respiratory rate 16 breaths/min. There was no lymphadenopathy, and jugular venous pressure was not elevated. Visual acuity was 6/6, with normal visual fields and perimetry. Fundoscopy was normal. Complete blood count, liver function tests, renal function tests, erythrocyte sedimentation rate, antineutrophil antibodies, electrocardiography, transthoracic echocardiography, carotid Doppler, and MRI of the brain with contrast were all normal. She is taking no medications.
Flickering sensation in eyes
The American Diabetes Association (ADA) position statement on diabetic retinopathy states that hyperglycemia has been the most consistently associated risk factor for retinopathy. A large and consistent set of observational studies and clinical trials confirms the association of poor glucose control and retinopathy.
The Diabetes Control and Complications Trial (DCCT), a randomized controlled clinical trial of intensive glycemic control vs conventional glycemic control in people with type 1 diabetes (T1D), demonstrated that intensive therapy reduced the development or progression of diabetic retinopathy by 34%-76%. The DCCT also demonstrated a definitive relationship between hyperglycemia and diabetic microvascular complications, including retinopathy. Early treatment with intensive therapy was effective.
The UK Prospective Diabetes Study (UKPDS) of patients with newly diagnosed T2D conclusively demonstrated that improved blood glucose control reduced the risk for retinopathy and nephropathy and, possibly, neuropathy. The overall microvascular complication rate was decreased by 25% in patients receiving intensive therapy vs conventional therapy. Epidemiologic analysis of the UKPDS data showed a continuous relationship between the risk for microvascular complications and glycemia, such that every percentage-point decrease in A1c (eg, 9% to 8%) was associated with a 35% reduction in the risk for microvascular complications.
More recently, the ACCORD trial of medical therapies demonstrated that intensive glycemic control reduced the risk for progression of diabetic retinopathy in people with T2D of 10 years' duration. This study included 2856 ACCORD participants enrolled in the ACCORD Eye Study and followed for 4 years.
The ADA recommends screening by an ophthalmologist for diabetic retinopathy within 5 years of the diagnosis of T1D and at the time of diagnosis of T2D. Women with preexisting diabetes who are planning pregnancy or who have become pregnant should be screened before pregnancy or in the first trimester.
While optimization of blood glucose, blood pressure, and serum lipid levels in conjunction with appropriately scheduled dilated eye examinations can substantially decrease the risk for vision loss from diabetic retinopathy, a significant proportion of those affected with diabetes develop diabetic macular edema or proliferative changes that require intervention. ADA treatment recommendations are:
• Refer patients with any level of macular edema, severe nonproliferative diabetic retinopathy (a precursor of proliferative diabetic retinopathy), or proliferative diabetic retinopathy to an ophthalmologist knowledgeable and experienced in the management and treatment of diabetic retinopathy.
• Laser photocoagulation therapy reduces the risk for vision loss in patients with high-risk proliferative diabetic retinopathy and, in some cases, severe nonproliferative diabetic retinopathy.
• Intravitreous injections of anti–vascular endothelial growth factor are indicated for central-involved diabetic macular edema, which occurs beneath the foveal center and may threaten reading vision.
Romesh K. Khardori, MD, PhD, Professor, Department of Internal Medicine, Division of Diabetes, Endocrine, and Metabolic Disorders, Eastern Virginia Medical School; EVMS Medical Group, Norfolk, Virginia
Romesh K. Khardori, MD, PhD, has disclosed no relevant financial relationships.
Image Quizzes are fictional or fictionalized clinical scenarios intended to provide evidence-based educational takeaways.
The American Diabetes Association (ADA) position statement on diabetic retinopathy states that hyperglycemia has been the most consistently associated risk factor for retinopathy. A large and consistent set of observational studies and clinical trials confirms the association of poor glucose control and retinopathy.
The Diabetes Control and Complications Trial (DCCT), a randomized controlled clinical trial of intensive glycemic control vs conventional glycemic control in people with type 1 diabetes (T1D), demonstrated that intensive therapy reduced the development or progression of diabetic retinopathy by 34%-76%. The DCCT also demonstrated a definitive relationship between hyperglycemia and diabetic microvascular complications, including retinopathy. Early treatment with intensive therapy was effective.
The UK Prospective Diabetes Study (UKPDS) of patients with newly diagnosed T2D conclusively demonstrated that improved blood glucose control reduced the risk for retinopathy and nephropathy and, possibly, neuropathy. The overall microvascular complication rate was decreased by 25% in patients receiving intensive therapy vs conventional therapy. Epidemiologic analysis of the UKPDS data showed a continuous relationship between the risk for microvascular complications and glycemia, such that every percentage-point decrease in A1c (eg, 9% to 8%) was associated with a 35% reduction in the risk for microvascular complications.
More recently, the ACCORD trial of medical therapies demonstrated that intensive glycemic control reduced the risk for progression of diabetic retinopathy in people with T2D of 10 years' duration. This study included 2856 ACCORD participants enrolled in the ACCORD Eye Study and followed for 4 years.
The ADA recommends screening by an ophthalmologist for diabetic retinopathy within 5 years of the diagnosis of T1D and at the time of diagnosis of T2D. Women with preexisting diabetes who are planning pregnancy or who have become pregnant should be screened before pregnancy or in the first trimester.
While optimization of blood glucose, blood pressure, and serum lipid levels in conjunction with appropriately scheduled dilated eye examinations can substantially decrease the risk for vision loss from diabetic retinopathy, a significant proportion of those affected with diabetes develop diabetic macular edema or proliferative changes that require intervention. ADA treatment recommendations are:
• Refer patients with any level of macular edema, severe nonproliferative diabetic retinopathy (a precursor of proliferative diabetic retinopathy), or proliferative diabetic retinopathy to an ophthalmologist knowledgeable and experienced in the management and treatment of diabetic retinopathy.
• Laser photocoagulation therapy reduces the risk for vision loss in patients with high-risk proliferative diabetic retinopathy and, in some cases, severe nonproliferative diabetic retinopathy.
• Intravitreous injections of anti–vascular endothelial growth factor are indicated for central-involved diabetic macular edema, which occurs beneath the foveal center and may threaten reading vision.
Romesh K. Khardori, MD, PhD, Professor, Department of Internal Medicine, Division of Diabetes, Endocrine, and Metabolic Disorders, Eastern Virginia Medical School; EVMS Medical Group, Norfolk, Virginia
Romesh K. Khardori, MD, PhD, has disclosed no relevant financial relationships.
Image Quizzes are fictional or fictionalized clinical scenarios intended to provide evidence-based educational takeaways.
The American Diabetes Association (ADA) position statement on diabetic retinopathy states that hyperglycemia has been the most consistently associated risk factor for retinopathy. A large and consistent set of observational studies and clinical trials confirms the association of poor glucose control and retinopathy.
The Diabetes Control and Complications Trial (DCCT), a randomized controlled clinical trial of intensive glycemic control vs conventional glycemic control in people with type 1 diabetes (T1D), demonstrated that intensive therapy reduced the development or progression of diabetic retinopathy by 34%-76%. The DCCT also demonstrated a definitive relationship between hyperglycemia and diabetic microvascular complications, including retinopathy. Early treatment with intensive therapy was effective.
The UK Prospective Diabetes Study (UKPDS) of patients with newly diagnosed T2D conclusively demonstrated that improved blood glucose control reduced the risk for retinopathy and nephropathy and, possibly, neuropathy. The overall microvascular complication rate was decreased by 25% in patients receiving intensive therapy vs conventional therapy. Epidemiologic analysis of the UKPDS data showed a continuous relationship between the risk for microvascular complications and glycemia, such that every percentage-point decrease in A1c (eg, 9% to 8%) was associated with a 35% reduction in the risk for microvascular complications.
More recently, the ACCORD trial of medical therapies demonstrated that intensive glycemic control reduced the risk for progression of diabetic retinopathy in people with T2D of 10 years' duration. This study included 2856 ACCORD participants enrolled in the ACCORD Eye Study and followed for 4 years.
The ADA recommends screening by an ophthalmologist for diabetic retinopathy within 5 years of the diagnosis of T1D and at the time of diagnosis of T2D. Women with preexisting diabetes who are planning pregnancy or who have become pregnant should be screened before pregnancy or in the first trimester.
While optimization of blood glucose, blood pressure, and serum lipid levels in conjunction with appropriately scheduled dilated eye examinations can substantially decrease the risk for vision loss from diabetic retinopathy, a significant proportion of those affected with diabetes develop diabetic macular edema or proliferative changes that require intervention. ADA treatment recommendations are:
• Refer patients with any level of macular edema, severe nonproliferative diabetic retinopathy (a precursor of proliferative diabetic retinopathy), or proliferative diabetic retinopathy to an ophthalmologist knowledgeable and experienced in the management and treatment of diabetic retinopathy.
• Laser photocoagulation therapy reduces the risk for vision loss in patients with high-risk proliferative diabetic retinopathy and, in some cases, severe nonproliferative diabetic retinopathy.
• Intravitreous injections of anti–vascular endothelial growth factor are indicated for central-involved diabetic macular edema, which occurs beneath the foveal center and may threaten reading vision.
Romesh K. Khardori, MD, PhD, Professor, Department of Internal Medicine, Division of Diabetes, Endocrine, and Metabolic Disorders, Eastern Virginia Medical School; EVMS Medical Group, Norfolk, Virginia
Romesh K. Khardori, MD, PhD, has disclosed no relevant financial relationships.
Image Quizzes are fictional or fictionalized clinical scenarios intended to provide evidence-based educational takeaways.
A 48-year-old Black man with type 2 diabetes (T2D) presented with complaints of a "flickering" sensation and a decrease in brightness of colors in both eyes as well as floaters in his left eye for several weeks. He reported that his symptoms fluctuate with changes in his blood glucose levels. His last eye examination was 2 years ago and his ocular history was unremarkable. His medical history was significant with a history of hypertension and T2D requiring insulin. His most recent glycated hemoglobin (A1c), 2 months ago, was 8.4%. His BMI was 31.2. The patient's medications were dulaglutide 0.75 mg injection pen, glargine insulin 42 units, losartan 100 mg, and amlodipine 10 mg.
On examination, his best-corrected visual acuity was 20/20 in the right eye and 20/30 in the left eye. Confrontation fields were intact, extraocular movements were full and extensive, and both pupils were equal, round, and reactive to light without afferent pupillary defects. Anterior segment examination was unremarkable in both eyes, without iris neovascularization. Intraocular pressures were 17 mm Hg in the right eye and 16 mm Hg in the left eye. On dilated fundus examination, the cup-to-disc ratio was 0.45 horizontally and vertically, with the presence of 1/4 disc diameters of neovascularization of the disc in the right eye and 2/3 disc diameters of neovascularization of the disc in the left eye.
Posterior segment findings were significant for scattered microaneurysms and dot/blot hemorrhages in the maculae. In the periphery of both eyes, there were tortuous vessels, scatter microaneurysms with dot/blot hemorrhages, and multiple areas of neovascularization elsewhere, with several foci of vitreous traction. There was no vitreous hemorrhage or tractional retinal detachment of either eye.
Spectral domain optical coherence tomography revealed an epiretinal membrane in the right eye and a blunted foveal contour with parafoveal cystic spaces, probably secondary to vitreomacular contraction. The left eye also had an epiretinal membrane and blunted foveal contour secondary to vitreomacular adhesion. The patient was diagnosed with bilateral high-risk proliferative diabetic retinopathy.
Vitamins and CVD, cancer prevention: USPSTF says evidence isn’t there
The leading causes of death in the United States are cardiovascular disease (CVD) and cancer. CVD causes about 800,00 deaths per year (30% of all deaths), and cancer is responsible for about 600,000 deaths annually (21%).1 Many adults—more than 50%—report regular use of dietary supplements, including multivitamins and minerals, to improve their overall health, believing that these supplements’ anti-inflammatory and antioxidative properties help prevent CVD and cancer.2 However, this belief is not supported by evidence, according to the US Preventive Services Task Force.
What did the Task Force find? After a recent reassessment of the topic of vitamin and mineral supplementation, the Task Force reaffirmed its position from 2014: There is insufficient evidence to recommend the use of vitamins and minerals to prevent CVD and cancer.3
For most of the vitamins and minerals included in the systematic review—vitamins A, C, D, E, and K; B vitamins; calcium; iron; zinc; and selenium—the Task Force could not find sufficient evidence to make a recommendation. It is important to note that if any of these are taken at the recommended levels, there is no evidence of serious harm from using them.
However, there is good evidence to recommend against the use of beta carotene and vitamin E.3 For beta carotene, the harms outweigh the benefits: Its use is associated with an increase in the risk of CVD death, as well as an increased incidence of lung cancer in smokers and those with occupational exposure to asbestos. The evidence on vitamin E indicates that it simply does not provide any cancer or CVD mortality benefit.
How to advise patients. Both the US Department of Health and Human Services (DHHS) and the American Heart Association state that most nutritional needs can be met through the consumption of nutritional foods and beverages; supplements do not add any benefit for those who consume a healthy diet.4 The updated USPSTF recommendations support this approach for community-dwelling, nonpregnant adults.
For those adults who want to supplement their diets—or who need to do so because of an inability to achieve an adequate diet—urge them to follow the recommendations found in DHHS’s Dietary Guidelines for Americans, 2020-2025.4
1. Murphy SL, Xu J, Kochanek KD, et al. Deaths: final data for 2018. Natl Vital Stat Rep. 2021;69:1-83.
2. Cowan AE, Jun S, Gahche JJ, et al. Dietary supplement use differs by socioeconomic and health-related characteristics among US adults, NHANES 2011-2014. Nutrients. 2018;10:1114. doi: 10.3390/nu10081114
3. USPSTF. Vitamin, mineral, and multivitamin supplementation to prevent cardiovascular disease and cancer: US Preventive Services Task Force recommendation statement. JAMA. 2022;327:2326-2333. doi: 10.1001/jama.2022.8970
4. US Department of Agriculture and US Department of Health and Human Services. Dietary Guidelines for Americans, 2020-2025. Published December 2020. Accessed July 13, 2022. www.dietaryguidelines.gov/current-dietary-guidelines
The leading causes of death in the United States are cardiovascular disease (CVD) and cancer. CVD causes about 800,00 deaths per year (30% of all deaths), and cancer is responsible for about 600,000 deaths annually (21%).1 Many adults—more than 50%—report regular use of dietary supplements, including multivitamins and minerals, to improve their overall health, believing that these supplements’ anti-inflammatory and antioxidative properties help prevent CVD and cancer.2 However, this belief is not supported by evidence, according to the US Preventive Services Task Force.
What did the Task Force find? After a recent reassessment of the topic of vitamin and mineral supplementation, the Task Force reaffirmed its position from 2014: There is insufficient evidence to recommend the use of vitamins and minerals to prevent CVD and cancer.3
For most of the vitamins and minerals included in the systematic review—vitamins A, C, D, E, and K; B vitamins; calcium; iron; zinc; and selenium—the Task Force could not find sufficient evidence to make a recommendation. It is important to note that if any of these are taken at the recommended levels, there is no evidence of serious harm from using them.
However, there is good evidence to recommend against the use of beta carotene and vitamin E.3 For beta carotene, the harms outweigh the benefits: Its use is associated with an increase in the risk of CVD death, as well as an increased incidence of lung cancer in smokers and those with occupational exposure to asbestos. The evidence on vitamin E indicates that it simply does not provide any cancer or CVD mortality benefit.
How to advise patients. Both the US Department of Health and Human Services (DHHS) and the American Heart Association state that most nutritional needs can be met through the consumption of nutritional foods and beverages; supplements do not add any benefit for those who consume a healthy diet.4 The updated USPSTF recommendations support this approach for community-dwelling, nonpregnant adults.
For those adults who want to supplement their diets—or who need to do so because of an inability to achieve an adequate diet—urge them to follow the recommendations found in DHHS’s Dietary Guidelines for Americans, 2020-2025.4
The leading causes of death in the United States are cardiovascular disease (CVD) and cancer. CVD causes about 800,00 deaths per year (30% of all deaths), and cancer is responsible for about 600,000 deaths annually (21%).1 Many adults—more than 50%—report regular use of dietary supplements, including multivitamins and minerals, to improve their overall health, believing that these supplements’ anti-inflammatory and antioxidative properties help prevent CVD and cancer.2 However, this belief is not supported by evidence, according to the US Preventive Services Task Force.
What did the Task Force find? After a recent reassessment of the topic of vitamin and mineral supplementation, the Task Force reaffirmed its position from 2014: There is insufficient evidence to recommend the use of vitamins and minerals to prevent CVD and cancer.3
For most of the vitamins and minerals included in the systematic review—vitamins A, C, D, E, and K; B vitamins; calcium; iron; zinc; and selenium—the Task Force could not find sufficient evidence to make a recommendation. It is important to note that if any of these are taken at the recommended levels, there is no evidence of serious harm from using them.
However, there is good evidence to recommend against the use of beta carotene and vitamin E.3 For beta carotene, the harms outweigh the benefits: Its use is associated with an increase in the risk of CVD death, as well as an increased incidence of lung cancer in smokers and those with occupational exposure to asbestos. The evidence on vitamin E indicates that it simply does not provide any cancer or CVD mortality benefit.
How to advise patients. Both the US Department of Health and Human Services (DHHS) and the American Heart Association state that most nutritional needs can be met through the consumption of nutritional foods and beverages; supplements do not add any benefit for those who consume a healthy diet.4 The updated USPSTF recommendations support this approach for community-dwelling, nonpregnant adults.
For those adults who want to supplement their diets—or who need to do so because of an inability to achieve an adequate diet—urge them to follow the recommendations found in DHHS’s Dietary Guidelines for Americans, 2020-2025.4
1. Murphy SL, Xu J, Kochanek KD, et al. Deaths: final data for 2018. Natl Vital Stat Rep. 2021;69:1-83.
2. Cowan AE, Jun S, Gahche JJ, et al. Dietary supplement use differs by socioeconomic and health-related characteristics among US adults, NHANES 2011-2014. Nutrients. 2018;10:1114. doi: 10.3390/nu10081114
3. USPSTF. Vitamin, mineral, and multivitamin supplementation to prevent cardiovascular disease and cancer: US Preventive Services Task Force recommendation statement. JAMA. 2022;327:2326-2333. doi: 10.1001/jama.2022.8970
4. US Department of Agriculture and US Department of Health and Human Services. Dietary Guidelines for Americans, 2020-2025. Published December 2020. Accessed July 13, 2022. www.dietaryguidelines.gov/current-dietary-guidelines
1. Murphy SL, Xu J, Kochanek KD, et al. Deaths: final data for 2018. Natl Vital Stat Rep. 2021;69:1-83.
2. Cowan AE, Jun S, Gahche JJ, et al. Dietary supplement use differs by socioeconomic and health-related characteristics among US adults, NHANES 2011-2014. Nutrients. 2018;10:1114. doi: 10.3390/nu10081114
3. USPSTF. Vitamin, mineral, and multivitamin supplementation to prevent cardiovascular disease and cancer: US Preventive Services Task Force recommendation statement. JAMA. 2022;327:2326-2333. doi: 10.1001/jama.2022.8970
4. US Department of Agriculture and US Department of Health and Human Services. Dietary Guidelines for Americans, 2020-2025. Published December 2020. Accessed July 13, 2022. www.dietaryguidelines.gov/current-dietary-guidelines